SlideShare una empresa de Scribd logo
1 de 382
COMMAND HOSPITAL PUNE
          25TH July 2008
INTRODUCTION to
Objective structured clinical examination

        Observation Stations &
         Clinical Examination

              5 min each
OBSERVED
                                       STATION




             OBSERVED
             STATION




                            OBSERVED
                            STATION
  OBSERVED                             REST
  STATION                              STATION




             REST STATION




STATION 1    OBSERVED                  REST STATION
             STATION
G        cen
  AB               ario
     G
 Kar
    y
Bio otype                 Blank OSCE Sheets
   sta
       tisti
             cs
1.    STATIONS:
2.    OBSERVED STATION: Foreign Body Apiration: demonstrate action on a manikin
3.    Statistics
4.    Neonatal Chest X ray with MAS and umbilical venous cannulation. Qs on complications and heparin infusion
5.    Rest Station
6.    Spirometry
7.    OBSERVED STATION: History Taking: examiner is observing your procedures.
8.    History taking of a 4 year old child with febrile seizures—have all the points to diagnose, prognosticate and manage the child
      covered.
9.    Congenital Anomaly: Qs on diagnosis, prognosis and causes of death.
10.   Photomicrograph of a PBS of a haemolytic anaemia. Qs on the abnormalities of the RBCs, the diagnosis, two inv, two treatment
      stratigies.
11.   X- Ray Chest of a CHD. Qs on radiological findings, diagnosis, antenatal history, DD, definitive manangement of the condition.
12.   Cavernous sinus thrombosis
13.   OBSERVED STATION: drug counselling to the mother of a child with bronchial asthma: from the drugs to devices.
14.   Photomicrograph of a PBS of different types of mononuclear WBCs. Qs on identification of the cells.
15.   Photograph of a dermatological condition. Qs on diagnosis, etiology, management.
16.    Vaccine: Qs on a particular vaccine, its storage, route, indications.
17.    Drug: Qs on the mechanism of action, group, metaboism, SEs.
18.    OBSERVED STATION: Examination of a system.
19.    Photograph of a Karyotype. Qs on the abnormality, diagnosis, 3 CFs, type of inheritance.
20.    Micronutrients and their deficiency disorders.
21.    Biostatics: calculation of relative risk, Odd’s ratio and Attributable risk.
22.    Photograph of an ectoparasite. Qs on identification, disease caused, drug therapy.
23.    ECG: abnormality, axis, diagnosis, complications, management.
24.    OBSERVED STATION: examiner asses the skills. Newborn gestational age assessment.
25.   REST STATION
26.    Neonatal: HMD:Qs four risk factors, ventilation modalitiws, ABG: a/A ratio, A-aDO2, OI for the ABG & ventilator settings.
27.    Lab Test: Haemolytic anaemia, antenatal diagnosis, management.
28.    Short Stature scenario with Ht, Wt, SMR, Bone age given. Qs on endocrine inv.
29.    Drugs: doses, routes, antidotes.
30.    Clinical Situation: GBS
31.    Photograph of oxygen delivery devices: Qs on Fio2 rates delivered.
32.

       REST STATION

X-Rays/CT/MRI
If patient presents with fever and toxaemia, give three
 differential diagnosis
 Give three modalities of management
1.   Lung Abcess
2.   Infected Bronchogenic cyst
3.   Infected Hydatid Cyst

1. Antibiotics (anaerobic +aerobic)
2. Chest physiotherapy
3. Percutaneous CT guided aspiration



                                       0.5 each
                                       Total 3 marks
What is the lesion ?
Give two management modalities.
What complication can occur during management &
 how can it be prevented?
NCC                          1
NCC:
Anticonvulsants              1
Albendazole

Increase in Cerebral Edema   1
Use steroid cover


                                  Total 3 marks
Describe the lesion.
What are the management modalities of such lesions
 especially if they are leading to complications ?
Haemangioma scalp                                    1

Conservative                                         2
Oral steroids
Local steroids
PDL: pulse dyed laser
IFN alfa interferon (in case of haemangiomas causing
 pressure effects or erosion leading to significant
 morbidity)




                                             Total 3 marks
Worrisome Hemangiomas
  Multiple cutaneous hemangiomas: Associated with
   visceral hemangiomas (e.g., liver)
  Large hemangiomas: May cause significant disfigurement of
   underlying structures and may be associated with congestive
   heart failure
  "Beard" hemangiomas: May be a marker for underlying
   laryngeal or subglottic hemangioma that may impair
   respiratory function
  Midline spinal hemangiomas: May be a marker for
   underlying spinal cord abnormality
  Head and neck hemangiomas: May be associated with
   other congenital anomalies, including central nervous system,
   cardiac, ocular, and sternal defects (e.g., Posterior fossa
   malformation, Hemangioma, Arterial abnormalities,
   Coarctation, Eye abnormalities, Sternal defects [PHACES]
   syndrome).
  Vulnerable anatomic locations: Impair vital functions,
   cause disfigurement (e.g. periocular, neck, lip, nasal tip)
  Ulcerated hemangiomas: Increased risk of superinfection,
   cause pain and lead to scarring
What are two imp radiological abnormalities?
What is the diagnosis ?
What are the three imp investigations ?
What are the metabolic abnormalities expected?
What is the management ?
 Distension of stomach
 No gas in the intestines

 Pyloric Stenosis

 USS abdomen: pyloric thickness >4mm/ pyloric
  length >14 mm
 S electrolytes
 Ba studies

 Hypochloremic, hypokalemic, metabolic alkalosis

 Management of the fluid & electrolytes
 Ramstedt’s pyloromyotomy

                                           0.5 each
                                           Total 5 marks
What is the diagnosis ?
What is the likely organism ?
Pnematoceles                             1

Likely organism Staphylococcus
       1




                                 Total 2 mark
Give five radiological findings ?
What is the diagnosis ?
What is the management ?
1 Ground glass appearance of bone 2
2 Thinned cortex
3 Periosteal calcification
4 White line of Fraenkel (well calcified cartilage)
5 Wimberger’s sign (white ring)


Scurvy      1

1 Vit C 100-200 mg/ day1
2 Dietary Therapy



                                             Total 4 marks
What was the lesion ?
What was the procedure carried out ?
What are the complications ?
VSD      1
Device implant 1


Device displacement   2
Emboli formation
Haemolytic Anaemia




                           Total 3 marks
Name the Investigation ?
What is the diagnosis ?
What are the further investigations ?
MCU


Posterior urethral Valves


DTPA (diethylene tetra amine pentoic acid) Function
DMSA (dimercapto succinic acid) -scarring




                                         Total 3 marks
What is the abnormality?
What is the diagnosis ?
How is it suspected clinically ?
What is the management ?
1.   .Bowelloops in lt hemithorax, mediastinal shift to right   1
2.   Congenital diaphragmatic hernia                            1

3.   Respiratory distress
     Mediastinal shift
     Bowel sounds in the thorax
      Scaphoid abdomen                                              2

4.   Avoid B&M vent
     prop up and decompress stomach
     Ventilate
     Treat PPHN
     Surgical correction
     Antenatal tracheal ligation                                3
CDH
  Pleuroperitoneal membrane defect
Fetal lung development
  Pseudoglandular stage      5 to 16 weeks
  Canalicular                16 to 24 weeks
  Saccular stage
  Alveolar stage             continues till 8 years of life
What is the abnormality?



What is the likely diagnosis?



 Delineate management.

 Mention 3 complications
Multiple fluid levels.
     gasless lower abdomen                                 1
    Small bowel obstruction                                1
    Surgical correction                                    1
    Dyselectrolytemia                                     1
     Perforation
     Exaggerated hyperbilirubinemia
     Dysmotility syndrome

Martin: Fanaroff and Martin's Neonatal-Perinatal Medicine, 8th ed.,
Describe the X –ray
Diagnosis
Give three recent advances in management
Hyper inflated lung fields with areas of atelectasis interspersed
  with areas of overinflation                                1
MAS                                                 1
Lung lavage
 surfactant instillation
 HFOV
 NO
 liquid ventilation                                  2
What is the abnormality
Three high risk situations when this condition is
 imminent
Management
Pneumothorax                  1
CDH
 Ventilation
 Post surfactant ventilation   2
Chest tube drainage           1
What is the abnormality
What is it a complication of
Pneumomediastinum      1
Forceful ventilation   1
Identify the abnormality
Clinical presentation
Treatment
Pneumopericardium        1
Shock with weak pulses       1
Drainage                 1
What is the diagnosis ?
1 mark
What are the MRI findings ?
What is the DD
What further inv will you do on the CSF to confirm
 the diagnosis ?
Acute disseminated encephalomyelitis




                3 marks
Oligoclonal bands are also found in:
 Multiple sclerosis
 Devic's disease
 Systemic lupus erythematosus
 Neurosarcoidosis
 SSPE
 Subarachnoid haemorrhage
 Syphilis
 CNS Lymphoma
What are the MRI findings ?
What could be the etiology ?
Multifocal cerebral & cerebellar encephalomalacia
Sequel of ischaemic injury in the perinatal period.




                                       2 marks
What is the likely diagnosis ?
What is the IP?
Which is the commonest site?
TB Spine                  1
2 YRS                      1
Thoraclumbar
  area of max stress
  valveless venous drainage 2




            4 mark
What are the MRI
 findings ?
Likely etiology
Likely CSF picture
TBM




Increased proteins
                                      3 mark
 Increased cells likely lymphocytes
 Low sugar
What is the X ray suggestive of ?
What is the likely organism?
What are the complications?
What is the drug of choice?
What is the duration of therapy?
Lobar Pneumonia
Pnemococous/Staphylococcus
Synpneumonic effusion, empyema,
Penicillin for susceptible org and cefotaxime /
 vancomycin for penicillin resistant org
10-14 d



                                                   5 marks
Questions
1.   What is the anatomical structure in which coin is
     lodged?
2.   What is the location of carina with respect to
     thoracic vertebrae?
3.   What are the anatomical areas of esophageal
     narrowing?
4.   How can this foreign body be removed?
Answers                                                 1

Esophagus
 When foreign bodies lodge in the esophagus, the flat
 surface of the object is seen in the AP view *
T 4                                                    1

Anatomic areas of esophageal narrowing                 3
  Cricoid
  Tracheal bifurcation
  Gastro-esophageal junction
Endoscopy
                                                        1
                                               *
L
Question
1.    What is the diagnosis?
2.    Fill in the blanks
     In an exudative pleural fluid
        a.   Proteins >
        b.   Pleural Fluid LDH > ………….
        c.   Fluid to serum LDH ratio > ………..
        d.   Cell count ………
3.    What does VATS stand for?
Answers                                       2
1.    Pleural Effusion (Right)
2.    Fill in the blanks
     In an exudative pleural fluid
                                              0.5
        a.   Proteins > 3.0 g/dL
                                              0.5
        b.   Pleural Fluid LDH > 200 IU/L
        c.   Fluid to serum LDH ratio > 0.6   0.5
        d.   Cell count > 1000                0.5
3.    Video Assisted Thoracoscopic Surgery    1
Questions
1.   Diagnosis?
2.   What are the first two steps in treatment of hypoxic spell?
3.   In a cyanotic newborn, how can you distinguish pulmonary
     disease from cyanotic congenital heart disease?
4.   Which cardiac conditions are associated with following
       a.   Egg Shaped Heart
       b.   Snowman silhouette
       c.   Rib notching
Answers                                                         1.5
1.    Cyanotic Congenital Heart Disese
      Probably TOF
      The heart size is normal
      Pulmonary vascular markings are decreased
      A hypoplastic main pulmonary artery segment
       contributes to the formation of the “boot-shaped”
       heart.




                                              Pediatric cardiology
                                              Myung K Park 5th ed
Answers                                                                           1
2.    Knee Chest Position                                                         1
      Morphine
                                                                                  1
3.    Hyperoxia Test
4.    X-ray appearances
     a.   Egg Shaped Heart                  Transposition of great arteries      0.5
     b.   Snowman silhouette       Total anomalous pulmonary
                                            venous return (supracardiac)         0.5
     c.   Rib notching                      Co-arctation of aorta (long
                                      standing)                                  0.5




                               Park: Pediatric Cardiology for Practitioners, 5th ed.
3 day neonate with
Lethargy
Feed refusal
Abdominal distension
Questions
1.   What stage of NEC is depicted in the
     X-ray?
2.   What is the radiological feature of Bell stage III
     NEC?
3.   Name two more conditions associated with
     pneumatosis intestinalis?
Answers                                                            1
    1.   NEC Stage II                                                 1
    2.   Pneumoperitoneum
                                                                      2
    3.   Any two of following
         Hirschsprung's disease, Pseudomembranous
         enterocolitis, Neonatal ulcerative colitis,
         Ischemic bowel disease




PART 5 Neonatal Necrotizing Enterocolitis
Martin: Fanaroff and Martin's Neonatal-Perinatal Medicine, 8th ed.,
6 weeks infant
Case of Cholestatic jaundice
 (Extra-Hepatic Biliary Atresia)
c/o swelling left thigh
Questions
1.   What is the likely cause of fracture femur in this
     case?
2.   How can this complication be prevented?
3.   How do you manage pruritus in these patients?
4.   An infant with cholestasis, triangular facies, and a
     pulmonic stenosis murmur is likely to have what
     syndrome?
Answers                                               1
1.   Metabolic Bone disease (secondary to Vitamin D
     deficiency due to malabsorption of fat soluble
     vitamins)
                                                      1
2.   Replace 5,000-8,000 U /d of D2,or
     3 -5 µg/kq/d of 25-hydroxycholecalciferol
3.   Ursodeoxycholic acid 15-20 mg/kg/day             1
4.   Alagille syndrome
                                                      1
     (Arteriohepatic dysplasia)
L   Previously healthy 7 years
         girl c/o
c
        Sudden onset weakness right
         upper and lower limb
t       Facial palsy right (UMN)
        Normal sensorium
        No fever/ trauma/ seizures
Questions
1.   What is the level of lesion on MRI?
2.   What are the structures marked
     c
     t
3.   Which hemoglobinopathy can be associated with
     this kind of presentation?
4.   A dilated and unreactive pupil indicates the
     compression of what structure?
Answers                                                      2
1.   Infarct in the left basal ganglia, the posterior limb
     of internal capsule, and the head of the caudate        1
Answers
          2.   C         Caudate           0.5
                         nucleus
               T         Thalamus
                                           0.5
               P         Putamen
               G         Globus pallidus

               White arrows indicate the ant
               and post limbs of internal
               capsule


                                           1
Answers                                 1
                                            1
3.   Sickle cell anemia
                                        1
4.   Compression of 3rd cranial nerve       1
Questions?
1.   Diagnosis?
2.   What is the treatment (mention complete
     schedule)?
Answers
                            1
1.   Miliary tuberculosis
2.   2HRZE + 7HR            1
     IAP Group 4
1.   Diagnosis
2.   What are the
     embryologic events that
     lead to this
     development?
3.    What are three causes
     of respiratory distress in
     a baby born with this
     condition?
Answers                                                            1
1.   Congenital Diagphragmatic Hernia
2.   The posterolateral portion of the diaphragm has remained      1
     open between the ninth and tenth weeks of gestation as a
     result the viscera will pass into the chest, and a CDH will
     result.
3.   a) Mechanical compression of the lungs from the herniated
                                                                       1
     viscera
     b) Pulmonary hypoplasia from compression of the
     developing lungs in utero                                         1
     c) Pulmonary hypertension
                                                                   1
Questions
Describe the lesion?
Give two D/D
What is the triad of tumor lysis syndrome?
Answers                             1
Osteolytic lesion of skull
                                    2
Histiocytosis
 Ewing`s Sarcoma
 Lymphoma                           3
 Bone cyst
Hyperuricemia, hyperkalemia, and
 hyperphosphatemia
Questions
1.   Describe the X-ray appearance
2.   Pathogenesis of the appearance
3.   Possible Diagnosis
4.   Which disorder is most commonly associated with
     an elevated MCHC?
5.   How is the corrected reticulocyte count calculated?
Answers
1. Sunray appearance                               1

2.   Medullary widening                            1

3.   Chronic hemolytic anaemia                     1
4.   Hereditary spherocytosis
                                                   1
5.   Corrected retic count
     = reticulocyte % × (patient Hct/normal Hct)       1
Below is a midline sagittal cut of a MRI scan of the
brain. View the midline anatomic diagram of the brain
and identify the following structures
Answer
S            -   Suprasellar cistern
P0           -   Pons
P            -   Midbrain (cerebral peduncles)
M            -   Medulla
C            -   Quadrigeminal plate (superior and inferior
 colliculi)
Q            -   Quadrigeminal cistern
V            -   Fourth ventricle




                                             1 mark each
Question
1.   Diagnosis?
2.   What is the emergency management of the
     condition?
3.   What is subsequent management after the
     emergency management is over?
Answers                                            1
1.   Pneumothorax,                                 1
     with mediastinal shift
                                                   1
2.   Put in a needle in second intercostal space
3.   Intercostal drain
                                                       1
Questions
1.   What is the diagnosis?
2.   Describe three features seen on the X-ray of the
     disease?
3.   What biochemical test would help clinch the
     diagnosis?
4.   What is the treatment of the condition?
Answers                                              1

1.   Rickets                                         0.5

2.   a) Cupping                                  0.5
     b) Widening of distal end of metaphysis
                                                     0.5
     c) Fraying
3.   Calcium, Phosphorus, Alkaline phosphatase   1.5
4.   Injection Vitamin D 6 lac unit IM stat
     PO Calcium
                                                 1

                                                 1
Questions
1.   What is the diagnosis?
2.   What is the clinical sign for the diagnosis called as?
3.   Name one intervention which can lead to this?
Answers                                         1

1.   Pneumopericardium                          1
2.   Hammans sign
                                                1
3.   Invasive ventilation with high pressures
X-ray neck lateral
  view in a child
  with respiratory
  distress
Questions
1.   What is the diagnosis?
2.   Which is the commonest organism implicated in
     this disease?
3.   What antibiotics are useful in this condition?
Answers
1.   Epiglottitis                            1
2.   Hemophilus influenzae type B            1
3.   Cephalosporins/ Ampicillin/ sulbactam    1
Questions
1.   Diagnosis
2.   By what gestational age would this defect occur?
3.   This can be prevented in subsequent pregnancies
     by intake of Folic acid. Folic acid should be taken in
     what dose and started when?
Answers                        1

1.   Occipital Encephalocele   1
2.   26 days post conception
3.   Folic acid
     Dose: 0.4mg/day           1

     Periconceptional period   1
Questions
1.   Diagnosis
2.   What is the clinical
     picture?
3.   What is the requirement of
     echo before surgery?
Answers
    1.   Tracheo-esophageal fistula
    2.   Excessive drooling
         Respiratory distress
    3.   To rule out associated
         Congenital heart diseases
         Right sided aorta




PART 3 Selected Thoracic Gastrointestinal Anomalies
Martin: Fanaroff and Martin's Neonatal-Perinatal Medicine, 8th ed.,
Pictorials
What is the diagnosis ?
Name three complications.
What is the period of infectivity.
• Chicken pox                                          1
1. Pneumonia                                           1
2. Encephalitis
3. Ataxia
4. ADEM
5. Progressive varicella: organ invol, h’ge


• Airborne or direct contact : contagious 2 days before
  & till all lesions are dry                           1
                                              Total 3 marks
1.   Give the diagnosis.

2.   Give four important components.

3.   State the most important prognostic factors.
1.    Prune- Belly Syndrome         0.5
2.    Important components
     a.   Abnormal abdominal musculature        0.5
     b.   Abdominal cryptorchidism0.5
     c.   Renal and ureteric anomalies with VUR 0.5
     d.   Pulmonary hypoplasia      0.5
3.    Long term complication
     a.   ESRD 0.5
4.    Prognostic factors
     a.   Degree of renal dysplasia 0.5
     b.   Pulmonary hypoplasia`     0.5
          (Total marks 4.0)
1.   Give the diagnosis.

2.   State mode of inheritance.

3.   Name five important components.

4.   Name most frequent immunological defect.
1.    Ataxia – Telengiectasia               0.5
2.    Autosomal recessive                         0.5
3.    Important components
     a.   Cerebellar ataxia                 0.5
     b.   Oculo-cutaneous telengiectasia    0.5
     c.   Immuno-deficiency                 0.5
     d.   Sino-pulmonary infections         0.5
     e.   Lympho - reticular malignancies   0.5
4.    Selective absence of Ig A             0.5
                             (Total marks 4.0)
      (NO MARKS IF FIRST ANSWER IS WRONG)
1.Give the diagnosis


2
1.    Seckel Syndrome                          2.0
2.    Important features                       2.0
     a. Microcephaly
     b. Beak like nose “Bird face”
     c. Hypertelorism
     d. GU anomalies
     e. Growth retardation
                           (Total marks 4.0)
     (NO MARKS IF FIRST ANSWER IS WRONG)
1.   Give the diagnosis.

2.   Give three important complications.

3.   List three treatment modalities
1.Giant haemangioma   0.5
2.DIC 1.5
1. State the diagnosis
2. Name the abnormal karyotypes
3. List three prenatal diagnostic techniques
4. Indication for parental screening
1. Down Syndrome 0.5
2. Trisomy 21 1.5
   Translocation
   Mosaicism
3. Triple screen (b-HCG, a fetopr, estradiol)1.5
   Ultrasonography
   Fetal DNA analysis
   (maternal age >35 yrs)
4. Translocation0.5
   (Total marks 4)

     (No marks if first answer is wrong)
What is the condition?              1
Describe the lesions.               1
What is the mode of inheritance?   1
What is CNS association?            1
Tuberous sclerosis
Ash leaf macule , shagreen patch, angiofibroma
Autosomal dominant
Characteristic brain lesions are tubers located in
 convolutions of cerebral hemisphere typically in the
 subependymal region.
What is the condition?

How will you treat it?
Bitot’s spots


Vit A 6 mo to 1 year 1 lac U oral rpt 48 hrs
        1 yr to 6 years 2




                                                2 marks
What is this condition?
Name a metabolic complication arising
 from this condition
2 marks
Collodion baby (Ichthyosis)




Hypernatremic dehydration




                               2 marks
What is the
 abnormality seen?

What is the diagnosis?
What other
 investigation would you
 advise in this case?
Absence of depression of the right angle of mouth while
 crying
DAOM (deficiency of depressor anguli oris muscle)


2D echocardiography as the condition is associated
 with cardiac anomalies




                                                3 marks
QUESTIONS




What is the condition?
What is the root value of nerves involved?
What is the life threatening complication associated
 2 marks
Erb- Duchenne palsy




         C5,C6
        Diaphragmatic paralysis




                                                                    3 marks
Chapter 27 – Birth Injuries

Martin: Fanaroff and Martin's Neonatal-Perinatal Medicine, 8th ed
Name the condition.

What is the natural
 course of these lesions?
Strawberry angiomas


Lesions increase over the first few months of life and
 then regress




                                                 2 marks
Describe the
 abnormality 1

What is the likely
 diagnosis1
Webbed neck

  In a female
      Turner syndrome
  In a male
      Noonan’s Syndrome




                           2 marks
Questions
Diagnosis
Commonest differential
Management
Answers
    Epulis: granular cell tumor of gum
    Hemangioma
    Surgical excision
     Carbondioxide laser excision



                                                                    4 marks




Chapter 26 – Physical Examination of the Newborn
Martin: Fanaroff and Martin's Neonatal-Perinatal Medicine, 8th ed
Questions
Abnormalities
Syndrome
Inheritance
Answers
Proptosis ,high forehead,mid facial hypoplasia.
Crouzon’s syndrome.
Autosomal Dominant




                                         3 marks
What is the Diagnosis ?
What is the Differential Diagnosis?
What are the assoc anomalies?
What is the Management ?
1.   GASTROCHISIS                          1

    2.   OMPHALOCOELE                              1

    3.    Beckwith Wiedman
         Trisomies
         Congenital cardiac anomalies          1

    4.    Cover defect with sterile draping
         fluid replacement
         Early surgical correction             1



PART 4 Selected Abdominal Gastrointestinal Anomalies
Martin: Fanaroff and Martin's Neonatal-Perinatal Medicine, 8th ed
1.   Give the diagnosis

2.   Give five important components

3.   Name an important metabolic abnormality and its
     management
1.    Hemihypertrophy Syndrome                                    0.5
   2.    Important features
        a.   Macrosomia / (Hemi hypertrophy)                         0.5

        b.   Macroglossia                                            0.5
        c.   Hepato-splenomegaly                                     0.5
        d.   Omphalocele                                             0.5
        e.   Embryonal tumours                                       0.5
   3.    Metabolic abnormality & management
        a.   Hyperinsulism     Hypoglycaemia                         0.5
        b.   Diazoxide / pancreactectomy                             0.5
                                                      (Total marks 4.0)



Chapter 28 – Congenital Anomalies

Martin: Fanaroff and Martin's Neonatal-Perinatal Medicine, 8th ed
1.   Give the diagnosis.
2.   Name the most important endocrinal feature
3.   Name the most important cardiovascular problem
4.   Name the mainstay of management
1.   Congenital Lipodystrophy                         1.0
2.   Insulin resistant DM                       1.0
3.   Hypertrophic cardiomyopathy                1.0
4.   Dietary fat regulation                     1.0
                               (Totalmarks 4)
        (No marks if first answer is wrong)
1.   Give the diagnosis
2.   List clinical staging.
3.   What is the most important sequel and it’s
     prediction?
4.   Outline the treatment.
1.    Kawasaki Disease                                 0.5
2.    Clinical Staging
     a.   Acute febrile phase                          0.5
     b.   Sub-acute phase                                      0.5
     c.   Convalescent phase                           0.5
3.    Complication and prediction
     a.   Coronary artery aneurism                             0.5
     b.   Onset of coronary artery changes                     0.5
          within 2 months of onset of illness
1.    Treatment
     a.   IVIG                                         0.5
     b.   High dose aspirin                            0.5
                                                (Total marks 4.0)
          (NO MARKS IF FIRST ANSWER IS WRONG)
1.   What is the diagnosis?

2.   What is the embryologic basis
     of these defects?

3.   In an otherwise healthy child,
     when are these defects
     repaired?
Answers
1.   Bilateral Cleft Lip and Palate         1
2.   Cleft lip
     - Hypoplasia of mesenchymal layer       1 resulting in
     failure of medial nasal and maxillary process to join
     Cleft palate
     - Failure of palatal shelves to fuse 1
3.    Cleft lip             By 3 Months           1 Cleft
     palate          By one year          1
Questions
1.   What is the likely diagnosis?
2.   What is the mode of inheritance?
3.   State True or False
     a.   Limb shortening is greatest in
          proximal segments.
     b.   Fingers often have a trident
          configuration.
     c.   Lumbar canal stenosis generally
          does not develop till early
          adulthood.
Answers
1.    Achondroplasia       1
2.    Autosomal dominant   1
3.    True/ false
     a.   True             1
     b.   True             1
     c.   True             1
A 3 year girl presents with
Severe pruritus esp
During night and after
Taking bath with warm
Water. Pruritus is most
Significant in web spaces
Of hands and toes


Questions
1. Diagnosis
2. What is causative
   organism?
3. What is Topical Rx?
4. What is oral Rx?
Answers                     1

1.   Scabies                1

2.   Sarcoptes scabie       1

3.   Permethrin 5%
                        1
4.   Ivermectin
 An infant presents with periorificial and acral
    dermatitis, diarrhea, alopecia and nail dystrophy.
Questions
1.   Diagnosis?
2.   What laboratory test helps to clinch the diagnosis?
3.   What is the treatment?
4.   Name any three IEM which can have similar
     finding.
Answers                                                          1

1.   Acrodermatitis enteropathica                                1
2.   Low plasma zinc concentration
3.   PO Zinc (zinc sulfate, acetate,or gluconate) 50-150mg/day   1
4.   Any three of following
     Maple syrup urine disease,                                  3
     Organic aciduria,
     Methylmalonic acidemia,
     Biotinidase deficiency
     Essential fatty acid deficiency
Questions
1.   Name the neurocutaneous
     marker seen
2.   In NF-1 what is the diagnostic
     criteria with respect to this
     marker?
3.   Name a X-linked dominant
     neurocutaneous syndrome
4.   What are the three stages of
     syndrome in question 3?
Answers
1.        Café-au lait macule                                                    1
2.        Six or more CAL macules larger than 5 mm in greatest diameter in
          prepubertal

          And larger than 15 mm in greatest diameter in postpubertal
          individuals
                                                                             1
3.        Incontinentia pigmenti
4.        Stage 1-Vesicular stage
          Stage 2-Verrucous stage
          Stage 3-Pigmented stage


                                                                             1

                                                                                 1


                                                                                 1
Questions
        1.   What is exhibited in the
             photographs a and b?
        2.   Diagnosis?
a
        3.   What is the mode of
             inheritance?
        4.   What is the major ocular
             criteria for diagnosis?




    b
Answers                                   1
1.   a: Hypermobility of finger joints.   1
     b: Positive thumb (Steinberg) sign
                                          1
2.   Marfan syndrome
3.   Autosomal dominant                   1

4.    Ectopia lentis                      1
     (superior and temporal )
1.   Diagnosis
2.   What is the pattern of inheritance?
3.   What is the characteristic radiographic image called?
4.   What is the opthalmologic complication?
Answers                          1

1.   Sturge weber syndrome   1
2.   Sporadic
                             1
3.   Tram track appearance
                             1
4.   Bupthalmos
Questions
1.   What is the diagnosis?
2.   What are the three
     species associated with it?
3.   What are useful methods
     for diagnosing tinea
     infections?
Answers                                               1
1.   Tinea infection                                      1

2.   Trichophyton                                     1
     Microsporum                                      1
     Epidermophyton
3.   Potassium hydroxide (KOH) preparations           1
     Fungal culture of skin scrapings inoculated on
     DTM (dermatophyte test media)                        1
Questions
1.   What is the clinical condition called?
2.   Which is the commonest malignancy associated
     with this?
3.   Which chromosome is implicated in malignancy
     mentioned in question 2?
4.   What other malignant disorders are associated with
     the malignancy being discussed?
Answers                     1
1.   Leucocoria             1
2.   Retinoblastoma
                            1
3.   RB1 gene Chr 13q 14
                            1
4.   Osteosarcoma
     Soft tissue sarcomas   1

     Malignant melanoma     1
Examination



              Lax skin folds
Diagnosis?
Cutis laxa
LAB & OTHER INV
Lab – 1(Blood)
  • Age – 4 yrs
  • Hb – 7 g%
  • RBCs – Polychromatophilic reticulocytosis,
    many spherocytes, central pallor less
    conspicuous
  • WBC and platelet series- normal
  • Bone Marrow – Erythroid Hyperplasia
  • Questions
    – Differential diagnosis
    – Confirmatory test
Answers: Lab – 1(Blood)
•   Anemia
•   Increased retic count – Hemolysis
•   PBS – spherocytes, other series normal
•   Bone marrow – hyperplastic – hemolysis
•   D/D of hemolytic anemia with spherocytes
    –   Hereditary spherocytosis – osmotic fragility
    –   Autoimmune hemolytic anemia – coombs test
    –   Wilsons disease – serum ceruloplasmin
    –   Burns - History



                                                       Total 2 marks
Lab – 2 (Blood)
• Age – 3yrs
• Hb – 6 gm%
• TLC – 1500/cmm
• Platelets – 20,000/cmm
• Bone marrow – Cellularity reduced
• Questions
  – Diagnosis?
  – Name 2 infectious causes
Answers: Lab – 2 (Blood)
All three cell lines are depressed
Bone marrow depression
Diagnosis- Aplastic anemia
Causes
  CMV
  Hepatitis B
  HIV




                                      Total 2 marks
Lab – 3 (Urine)
 •Age – 9yrs
 •Appearance - Turbid
 •Sp Gr – 1019
 •Proteins – 4 +
 •RBCs – 6 to 10 per HPF, occasional RBC casts seen
 Questions
     – Diagnosis?
     – Causes?
Answers: Lab – 3 (Urine)
Sp Gr – normal
Nephrotic range proteinuria
Microscopic hematuria with RBC casts – glomerular
 pathology
Nephrotic syndrome with hematuria – Nephritic
 onset Nephrotic syndrome
Common causes – Idiopathic, SLE, Malaria, HSP,
 Hepatitis B


                                         Total 3 marks
Lab – 4 (CSF)
Age – 3 days term neonate
CSF –
   Appearance – clear
   Proteins – 10 mg/dl
   Glucose – 45 mg/dl ( Blood sugar – 80 mg/dl)
   Cells – WBC – 6/cmm pred neutros
Questions
  What is your interpretation
Answers: Lab – 4 (CSF)
Normal CSF in a neonate

 WBC              11+/-10 (90th centile-22)
 polys           2.2+/-3.8 % (90th centile- 6)
Protein          84+/-45 mg/dl
Glucose          46+/-10 mg/dl

Interpretation – normal CSF report


                                           Total 3 marks
Lab - 5 Peritoneal fluid
    • AGE 10 yrs
    • Peritoneal fluid
      –   Protein 4.5g/dl
      –   LDH 750 IU (90u/L)
      –   WBC 7500/cmm
      –   GLUCOSE 56mg/dl (Blood sugar 80mg/dl)
      Question
      – Diagnosis?
      – Treatment?
Answer : Lab - 5 Peritoneal fluid
     Normal value
                          Transudate               Exudate
    Sp Gr                 <1.016                   >1.106
    Protein               <3gm/dl                          >3gm/dl
    WBCs                  <1000/cmm                >1000/cmm
    RBCs                  <10,000/cmm              Variable
    Glucose               = serum         <serum
    pH                    7.4 to 7.5               <7.4

     Diagnosis
       Tuberculosis
     Treatment-
       ATT as per IAP consensus Group 4
       2 HRZE / 7 HR



                                                      Total 2 marks
1.   What is the diagnosis?
2.   What is the mutation that results in this disease?
3.   What are the three main categories of crises in patients with
     sickle cell disease?
Answers                                                                1

1.   Sickle cell disease
2.   On the beta chain, valine is substituted for                       1
     glutamic acid at position 6
3.   Aplastic crisis                                                    1

     Vaso-occlusive crisis                                             1
     Acute splenic sequestration
                                                                       1




                   Nathan and Oski's Hematology of Infancy and Childhood,
                   6th ed. 2003, pp 802-811
Questions
1.    Describe the cells seen.
2.    Name one condition each
      from following category in
      which these cells are seen
     a.   Neurological
     b.   Metabolic
     c.   Hepatic
     d.   Endocrinal
Answers                                                 1

1.    Acanthocytosis
2.    Conditions
                                                        1
     a.   Neurological Neuroacantocytosis
                                                        1
     b.   Metabolic          Abetalipoproteinemia
     c.   Hepatic            Severe liver dysfunction   1

     d.   Endocrinal Hypothyroidism                         1
What is this modality of investigation?
Name three common indications .
Answer: Spots: 1
Radionuclide bone scan
Indication
  Metastasis
  Osteomyelitis
  Stress fracture




                          Total 2 marks
What is this modality of investigation?
Name one common indication .
Answer : Spots: 2
Hepatobiliary scintigraphy
Indication
  Biliary atresia




                              Total 2 marks
What is the diagnosis ?
What are the cardiac conditions associated ?
What are the endocrinal conditions associated ?
Turner’s Syndrome

1. Bicuspid aortic valve
2. Co of aorta
3. AS
4. MVP
5. TAPVR

Hypogonadism
Hypothyroidism
Type-II DM


                           Total 4 marks
Questions
1.    What is the diagnosis?
2.    Give two sexual
      characters in this
      condition
3.    Give one cardiac
      condition that may be
      associated with this
      condition
4.    What would be the
      plasma levels of
     a.   FSH
     b.   LH
     c.   Testosterone
Answers                                       1
1.    Klinefelter Syndrome (47 XXY)
2.    Any two                                  2
      Lack secondary sexual characteristics
      Infertility, azoospermia
      Gynecomastia
      Testicular dysgenesis
3.    Any one
      Mitral valve prolapse (55%)             0.5
      Varicose veins (20-40%)
4.    Plasma levels of
     a.   FSH            Increased            0.5
     b.   LH             Increased
                                              0.5
     c.   Testosterone   Decreased
                                              0.5
Questions
a. Diagnosis
b. Commonest differential how do you distinguish
 between two?
c. Give 3 Steps of management
Answers
PSVT
Sinus tachycardia - there is beat to beat variability.
Vagal manoeuvres:
 Carotid Massage
 Ice packs over face
 Valsalva manouver


Adenosine drug of choice.


                                         4 marks
                                            4 marks
ECG
Questions
Abnormalities
Management
Which drug eliminates the causative agent
Answers
ECG changes of hyperkalemia
Calcium gluconate, sodabicarb, asthalin nebulisation,
 insulin
Kayexelate resin.




                                         4 marks
What disease does this PBS picture suggest ?
Spherocytosis   1
CASE SCENARIOS
Case
pH 7.51 (7.35-7.45)      This is the ABG of a 6 week
PO2 12 KPa (95 mmHg)       neonate admitted with
PCO2 4.7 KPa (35 mmHg)     projectile vomiting
Blood Urea 11 mmol/l
Sodium 131 mmol/l
Potassium 3 mmol/l
Chloride 83 mmol/l
Questions
1.   What type of alkalosis the patient has: metabolic/
     respiratory?
2.   What is the most likely diagnosis?
3.   What are the ultrasonographic criteria to diagnose
     the condition?
4.   What is the treatment of choice?
Answers
1.   Metabolic alkalosis
2.   Congenital hypertrophic pyloric stenosis
3.   Ultrasonographic diagnostic criteria
     a) Pyloric thickness>4mm or
     b) Pyloric length > 14mm
4.   Pyloromyotomy (Ramstedt)
This is blood glucose profile of a 10 year boy
with Diabetes. His Hb A1C is 12%. At that time
his treatment included:
Insulin          Morning           Evening
Regular          6U                3U
NPH              8U                5U
                               400                      Blood glucose profile
       Blood Glucose (Mg/dl)




                               350
                               300
                               250
                               200
                               150
                               100
                               50
                                 0
                                                                         t
                                     AM




                                                                                AM




                                                                                                             AM
                                            00


                                                   00


                                                           00


                                                                      gh




                                                                                       00


                                                                                              00


                                                                                                     00
                                          12


                                                 17


                                                         22




                                                                                     12


                                                                                            17


                                                                                                   22
                                                                   ni
                                  00




                                                                               00




                                                                                                            00
                                                                 id
                                                                M
                                6:




                                                                             6:




                                                                                                          6:
                                                                    TIME
                                                                2
Questions
1.   What changes would you institute in insulin
     dosage?
2.   Considering his Hb A1c over what period his
     glycaemic control is likely to be unsatisfactory?
3.   When do you start screening for diabetic
     retinopathy ?
4.   What is the preferred method for screening for
     diabetic retinopathy?
Answers                       1
1.        Increase
          morning regular
          insulin dose by 2
          Units               1
2.        3 months
3.        After 5 years in    1
          prepubertal
          children
          after 2 years in    1
          pubertal
          children            1
4.        Fundal
          photography
D/D of polyuria and polydipsia
Screening guidelines
1.   What is the diagnosis?
2.   What is the preferred treatment and dose?
3.   Give three alternative drugs for treatment.
Answers                                             1

    1.    Ventricular tachycardia
                                                       1
    2.    Cardioversion 0.5 to 1 joule/kg
    3.    Amiodarone                                   1

          Procainamide                                 1
          Lidocaine                                    1




Park: Pediatric Cardiology for Practitioners, 5th ed
How does SVT in children differ from physiologic
sinus tachycardia?
SVT typically has the following features:
  Sudden onset and termination rather than a gradual
   change in rate
  Persistent ventricular rate of >180 bpm
  Fixed or almost fixed RR interval on ECG
  Abnormal P-wave shape or axis or absent P waves
  Little change in heart rate with activity, crying, or
   breath holding
Case
A 5 year girl is brought after accidental ingestion of
 unquantified insecticide with salivation, lacrimation
 and miosis
Questions
1.   Name three muscarinic actions of OP poisoning
2.   What is the specific antidote?
Answers
                                                       3
1.   Increased oral and tracheal secretions, miosis,
     salivation, lacrimation, urination, vomiting,
     cramping, defecation, and bradycardia
2.   Atropine
                                                       1
OP poisoning effects
Muscarinic effects:
  Increased oral and tracheal secretions, miosis,
    salivation, lacrimation, urination, vomiting, cramping,
    defecation, and bradycardia; may progress to frank
    pulmonary edema
Central nervous system effects:
  Agitation, delirium, seizures, and/or coma
Nicotinic effects:
  Sweating, muscle fasciculation, and, ultimately,
    paralysis
Case
A 8 year old boy is admitted after consumtion of > 60
 tablets of ferrous sulfate kept at home
Questions
1.   Which stage of iron toxicity is free of any
     symptoms?
2.   What is the preferred method of gastrointestinal
     decontamination in patients with iron overdose?
3.   What is the antidote with dose?
4.   What metal intoxication can mimic Kawasaki
     disease?
Answers                                                                          1
1.   Stage 2 (6-24 hours): Iron silently accumulates in
     the mitochondria during this period, which is
     relatively free of symptoms.
2.   Whole-bowel irrigation
     -Syrup of ipecac is no longer recommended, and activated charcoal will      1
     not adsorb iron. Many adult-strength iron-containing pills are very large
     and are often too large for orogastric lavage
3.   Deferoxamine                                                                1
     Up to 15 mg/kg per hour via continuous IV infusion                          1
4.   Mercury
                                                                                 1
     Acrodynia
Iron Toxicity Stages
Stage 1 (0.5-6 hours): During this stage, iron exhibits a direct
 corrosive effect on the small bowel.
 Symptoms include nausea, vomiting, abdominal pain, and/or
 gastrointestinal hemorrhage.
Stage 2 (6-24 hours): Iron silently accumulates in the
 mitochondria during this period, which is relatively free of
 symptoms.
Stage 3 (4-40 hours): This phase is characterized by systemic
 toxicity with shock, metabolic acidosis, depressed cardiac
 function, and hepatic necrosis.
Stage 4 (2-8 weeks): During this phase, pyloric stenosis and
 obstruction can develop as a result of earlier local bowel
 irritation.
Disease
                      Present               Absent

                         a                    b
          Positive
Test
                         c                    d
         Negative




       •Sensitivity                       •Positive predictive value
       • T(+)/ D(+) = a/a+c               •D+/T+ = a/a+b


       •Specificity                       •Negative predictive value
       • T(-)/ D (-) = d/b+d              • D-/ T- = d/ c+d
Case
Analyse the table below

              Test           Has disease   No disease


              Positive       90            10

              Negative       10            90



1.   Calculate sensitivity
2.   Calculate specificity
Answer                 1

1.   Sensitivity 90%   1

2.   Specificity 90%
Case                     Urine            Pure
                                          growth
                                                    Multiple
                                                    growth
                         microscopy

On auditing urine
     growth, following   >50 WBC          95        15
     were the result
                         <50 WBC          10        200



1.     Calcute positive predictive value of urine
       microscopy
2.     Calculate negative predictive value urine
       microscopy
Answer             1

1.   PPV 95/110

                   1
2.   NPV 200/210
Case 8
A 13-year old boy develops right upper-quadrant pain
 and fever with chills and rigors.
An abdominal ultrasound reveals
  Hyperechoic liver parenchyma
  Dilatation of several intrahepatic bile ducts
  Few bilateral renal cysts.
Questions
1.   What is the Diagnosis?
2.   What is the cause of fever with chills in this case?
3.   What are the components of this
     disease/syndrome?
4.   Which gene has been implicated for the above?
Answers                             1

1.   Caroli’s syndrome              1
2.   Cholangitis                    1
3.   Intra-hepatic ductal ectasia
     AR Polycystic kidney Disease   1

4.   PKHD1 gene                     1
Case9
A 3 year old boy a case of steroid sensitive nephrotic
 syndrome in remission has been brought with
  Periorbital puffiness
  Proteinuria on dipstick evaluation at home
Questions
1.   What are the components of nephrotic syndrome?
2.   How do you define proteinuria of nephrotic range
     on dipstick?
3.   How do you define relapse?
4.   What is the treatment of relapse?
Answers
1.   Nephrotic Syndrome       0.5
      Heavy proteinuria      0.5
      Hypoalbuminemia
                              0.5
      Edema
      Hypercholesterolemia   0.5
Answers                                                                 1
2.   Urine protein 3+/4+ (on dipstick)
3.   Urine albumin                                                      0.5
        3+ or 4+ (or proteinuria >40 mg/m2/h)
                                                                        0.5
        for 3 consecutive early morning specimens
        having been in remission previously                             0.5
                                                                        0.5
2.   Prednisolone
        2 mg/kg/day (single or divided doses) until urine protein is   0.5
         trace or nil for 3 consecutive days
        Then 1.5 mg/kg on alternate days for 4 weeks
        Then discontinued                                              0.5
                                                                        0.5
Case10
A 12 year old girl a case of chronic kidney disease due
 to FSGS is evaluated in OPD for hypertension, poor
 growth and dyslipidemia.
Questions
1.    How do you determine GFR using serum creatinine with
      Schwartz formula?
2.    What is Stage II Chronic Kidney disease (CKD) in terms of
      GFR?
3.    State True/ False
     a.   Growth hormone is a mode of treatment for short stature due
          to CKD.
     b.   Anemia in CKD is caused primarily due to loss of ferritin in
          urine.
     c.   The most common form of renal osteodystrophy seen with
          CKD is high turnover bone disease.
Answers
1.    GFR(ml/min/1.73m2)
                                                             1
              = k x Height (cm)
           serum creatinine (mg/dL)
      where k is
      0.33 for low-birthweight infants younger than 1 yr,
      0.45 for term AGA infants younger than 1 yr,
      0.55 for children and adolescent females
      and 0.70 for adolescent males
Answers                                                                                 1
2.    Stage 2 CKD: GFR 60-89 ml/kg/1.73m2
3.    True/ False                                                                       1
     a.   True
          Children with CKD who remain <2 SD for height despite optimal medical
          support may benefir from rHuGH (0.05mg/kg/d)
     b.   False                                                                         1
          Anemia is primarily the result of inadequate erythropoietin production
     c.   True
          The most common finding in renal osteodystrophy is osteitis fibrosa cystica       1
          (high turnover bone disease secondary to hyperparathyroidism)
Answers
2.    Stage 2 CKD: GFR 60-89 ml/kg/1.73m2                                               1
3.    True/ False
     a.   True                                                                          1
          Children with CKD who remain <2 SD for height despite optimal medical
          support may benefir from rHuGH (0.05mg/kg/d)
     b.   False
          Anemia is primarily the result of inadequate erythropoietin production        1
     c.   True
          The most common finding in renal osteodystrophy is osteitis fibrosa cystica
          (high turnover bone disease secondary to hyperparathyroidism)                     1
What are the main causes of chronic renal
disease in children that result in renal
transplantation?

Obstructive uropathy
Aplastic/hypoplastic/dysplastic kidneys
Focal segmental glomerulosclerosis
Questions
1.   Define Time constant
2.   Two conditions with increased time constant
3.   Two conditions with decreased time constant
Answers
                                                              1

1.Time constant is the product of compliance and
  resistance(amount of time required for proximal
  airway pressure to equilibrate with alveolar pressure)
2.Asthma and bronchiolitis
3.Pneumonia and pulmonary edema                               1

                                                              1




                                 Nelson18th edition pg 1722
Questions
1.   Mechanism of grunting
2.   Significance of grunting
3.   Helpful in which conditions (name two)
Answers                                                        1

1.   Produced by expiration against a partially closed
     glottis expiration
                                                               1
2.   Attempt to maintain positive airway pressure
     during expiration for as long as possible.
3.   Alveolar diseases(pneumonia/pulmonary
     edema/HMD) and small airway                               1
     obstruction(bronchiolitis)


                                  Nelson18th edition pg 1724
Questions
1.   Significance of clubbing
2.   Two thoracic causes of clubbing
3.   Two extrathoracic causes of clubbing
4.   Sign associated with clubbing
Answers                                   1

1.   Chronic hypoxia                      1
2.   Small lung cancer and lung abscess
                                          1
3.   Cyanotic heart diseases and IBD
4.   Schamroth’s sign                     1
Case
The EEG record is from a 5 month infant who
 episodically raises his arms and then flexes his neck,
 trunk and hips. The episodes last a few seconds and
 end with a brief cry and return to normal posture.
The episodes occur in quick succession with several
 hours passing between each cluster of attacks.
EEG
Questions
1.   What does the EEG show?
2.   What syndrome is suggested by the history and
     EEG?
3.   Give one genodermatoses condition associated with
     this syndrome.
4.   What is the long term prognosis?
Answers                                                  1
1.   Hypsarrhythmia
        Large amplitude slow wave activity mixed with
         multifocal spikes and sharp waves
Answers                                          1
2.   Infantile spasms- West Syndrome             1
3.   Tuberous Sclerosis
4.   Treatment
                                                 1
         Unsatisfactory on most occasions
         AED`s       Vigabatrin
                      Benzodiazepines
                      (Clonazepam/ Nitrazepam)
         Steroids/ ACTH
What is the classic triad of
infantile spasms?
Spasms, hypsarhythmia, and developmental
 regression



Infantile spasms are known as West's syndrome, and
 the condition is named for the physician who first
 described the condition in his own son in 1841.
After what period can AEDs be
   safely discontinued?
    When the child is free of seizures for 2 years




Smith R, Ball R: Discontinuing anticonvulsant medication in children.
Arch Dis Child 87:259-260, 2002
Case 15
The examination of a newborn reveals a lump of soft
 tissue of size of 50 paise coin overlying the lower
 spine. There is no neurological deficit.
Questions?
1.   What is the likely diagnosis?
2.   What is likely risk of recurrence of disorder in
     future pregnancies?
3.   What intervention started at what time and for
     how long, can reduce the risk of intervention?
4.   What are the options for managing bladder
     incontinence in this condition?
Answers dysraphism
1. Occult spinal
                                                        1


2.   Risk of recurrence                                     1

     One affected child                3-4%
     Two affected Children             10%
3.   Intervention        Folic acid 400 mcg/d           1
     Started             Periconceptionally           0.5
                         (started before pregnancy)
      Continued          Till 12 wks of preg           0.5
Answers                                     2
4.    Bladder incontinence management
      Clean intermittent catheterization
      Artificial urinary sphincter
      Surgical urinary diversion
      Augmentation cystoplasty
What is the likelihood that a patient with
myelomeningocele will have
hydrocephalus?

Hydrocephalus is seen in 95% of children with
  thoracic or high lumbar myelomeningocele.

The incidence decreases progressively with more
  caudal spinal defects to a minimum of 60% if the
  myelomeningocele is located in the sacrum.
Case 16
A 6-month-old child was noted to be normal at birth,
 but over the ensuing months you have been
 somewhat concerned about his slowish weight gain
 and his mild delay in achieving developmental
 milestones.
The family calls you urgently at 7:00 A.M. noting that
 their child seems unable to move the right side of his
 body.
Questions?
1.   Which of the following conditions might explain this child’s
     condition?
       a. Phenylketonuria
       b. Homocystinuria
       c. Cystathioninuria
       d. Maple syrup urine disease
       e. Histidinemia
1.   Which one other systemic examination would you like to do?
2.   What investigation would confirm the etiology?
3.   What treatment would you institute as a long term measure?
Answers                                     1
1. Homocystinuria (b)
                                            1
2. Eye (Ectopia lentis)
3. Elevated plasma levels for homocystine   1

4. Vitamin B6 (200-1,000 mg/24 hr)          1

     Folic acid 1-5mg/24 hr
     Vit B 12 (1-2 g/24hrs)
Match the following

Match the following sign/symptom with the expected
 location of the lesion in the brain:
a. Deafferented pupil             Nystagmus.

b. Cerebellum                     Tonic deviation of the eye(s).

c. Ipsilateral cortex             Marcus Gunn pupil.

d. Midbrain tectum                Horner’s syndrome.

e Pons                            Midposition pupils

f. Cribriform plate of ethmoid    Pinpoint pupils
                                  Leakage of CSF from nose.
Answer
a. Deafferented pupil   Marcus Gunn pupil
b. Cerebellum           Nystagmus
c. Ipsilateral cortex   Tonic deviation of the eye(s)
d. Midbrain tectum      Midposition pupils
e. Pons                 Pinpoint pupils
f. Cribriform plate     Leakage of CSF from nose




                                                        3 marks
Match the following
                                  Thrombotic stroke.
a. Rete mirabile of Moyamoya.
b. Right-to-left cardiac shunt.   Embolic stroke.
c. Arteriovenous
   malformations.                 Intracerebral hemorrhage.
Answer
a.   Rete mirabile of      Thrombotic stroke
     Moyamoya.
b. Right-to-left cardiac   Embolic stroke
     shunt.
c. Arteriovenous           Intracerebral h`age
     malformations.
Case19
A 4-year-old child is observed to hold his eyelids open
 with his fingers and has drooping of eyes, especially in
 the evening. He has some trouble swallowing his
 food. He can throw a ball, and he runs well.
He undergoes EMG and is diagnosed as

MYASTHENIA GRAVIS
Questions?
1.   What is the characteristic EMG in Myasthenia?
2.   What is the chest X-ray finding in this condition?
3.   What is the clinical test for diagnosing myasthenia?
4.   Which antibiotics can worsen the condition?
5.   What drugs are used for treatment?
Answers                                                1
1.   Decremental response to repetitive stimulation
2.   Enlarged thymus                                    1
3.   Edrophonium test
                                                        1
4.   Aminoglycosides
5.   Neostigmine 0.4mg/kg PO 4-6 hrly                  1
                  0.04mg/kg IM 4-6 hrly
                                                        0.5

                                                      0.5
Fleisher GR, Ludwig S (eds): Textbook of Pediatric Emergency Medicine, 3rd ed.
Case 20
Examination of the cerebrospinal fluid of an 8-year-
 old, mildly febrile child with nuchal rigidity and
 intermittent stupor for 3 weeks shows the following:
  White blood cells 100/μL (all lymphocytes)
  Negative Gram stain
  Protein         750 mg/dL
  Glucose         25 mg/dL
Questions?
1.   What are the likely differential diagnosis?
2.   What drugs can be used to reduce intracranial
     tension?
3.   Which cranial nerve is involved in false localizing
     sign?
Answers                               1
1.   Tubercular/ Fungal meningitis    1
2.   Mannitol, glycerol
                                      1
3.   Sixth cranial Nerve (Abducens)
Case 21
At birth, an infant is noted to have an abnormal
 neurologic examination.
Over the next 1-2 weeks he develops severe
 progressive central nervous system degeneration, an
 enlarged liver and spleen, macroglossia, coarse facial
 features, and a cherry-red spot in the eye.
Question
1.   The laboratory finding likely to explain this child’s
     problem is
       a. Reduced serum hexosaminidase A activity
       b. Deficient activity of acid beta-galactosidase
       c. A defective gene on the X chromosome
       d. Complete lack of acid alpha-galactosidase activity
       e. Deficient activity of galactosyl-3-sulfate-ceramide sulfatase
1.   What is the diagnosis?
Answers                                                  1
1.   Deficient activity of acid beta-galactosidase (b)
                                                         1
2.   Type 1 GM1 gangliosidosis
22
For each description select the most likely diagnosis:
Clinical scenario
1.   Eye blinking or throat-clearing noises
     in an otherwise healthy 8-year old                 Diagnosis
     boy
2.   A 6-year-old boy with eye twitching      a. Transient tic disorder of
     and ecolalia                               childhood
3.   A 2-year-old infant who was born
     prematurely and is unable to walk or     b. Tourette syndrome
     talk                                     c. Cerebral palsy
4.   A 14-year-old girl with a history of
     precocious puberty who now               d. Tuberous sclerosis
     develops a large goiter
                                              e. McCune-Albright syndrome
5.   An infant with infantile spasms, a
     hypsarrhythmic EEG pattern, and
     ash-leaf depigmentation on her back
Answer
1.   Transient tic disorder of childhood
2.   Tourette syndrome
3.   Cerebral palsy
4.   McCune-Albright syndrome
5.   Tuberous sclerosis




                                           Total 5 marks
Case scenario 23
16 year boy followed up in your clinic for several
 years. His latest lung function tests are
FVC                85%
FEV1               57%
PEF                53%
FEV1/FVC           67%
Questions
1. Diagnosis
2. Which of the above measurement is best for
   monitoring
Answer
1. Asthma

2. FEV1/FVC
List the possible acute side effects
of salbutamol
General: Hypoxemia, tachyphylaxis
Renal: Hypokalemia
Cardiovascular: Tachycardia, palpitations,
 premature ventricular contractions, atrial fibrillation
Neurologic: Headache, irritability, insomnia, tremor,
 weakness
Gastrointestinal: Nausea, heartburn, vomiting
What proportion of childhood asthmatics
   "outgrow" their symptoms?
    30-50%




Sears MR, Greene JM, Willan AR, et al: A longitudinal, population-based,
cohort study of childhood asthma followed to adulthood.
N Engl J Med 349:1414-1422, 2003.
Question 24
Below is diagram of lung volumes,what do the letters
 A and B represent
Answer
A inspiratory reserve volume



B residual lung volume
Case No 28

  7 years male c/o increasing weakness of all limbs x
  3 days & URTI one week back
Examination
  CNS
  Bilateral facial nerve palsies
     Power Grade 0 to 1 both legs; grade 3 arms
     Loss of DTJ,No sensory deficit/ papilloedema
  Abdomen
     Palpable midline mass arising from pelvis
  Other systems: Normal
Questions
1.   What is the likely diagnosis?
2.   Mention 3 investigations to clinch diagnosis.
3.   What is the most sensitive measure of respiratory
     muscle involvement?
4.   Mention 3 modalities of treatment.
5.   What is the midline mass, explain its significance
     and how should it be managed?
Answers
1.    Guillain-Barre Syndrome                  (1)
      The history points to a symmetrical motor
       neuropathy without sensory nerve involvement.
      Muscle disorders should have normal DTJ
      Polio will cause asymmetrical weakness
      Spinal cord abnormalities will not cause facial
       muscle weakness
Answers
  2. CSF-protein cellular dissociation
      NCV
      EMG
  (1.5)
  3. The most sensitive measure of respiratory muscle involvement is (1)
    VITAL CAPACITY
      In children, the normal VC may be calculated as VC = 200 mL × age
      in years. If the VC falls below 25% of normal, endotracheal intubation
      is performed.
   4. IVIG,Plasmapheresis,steroids                                      (1.5)
   5. Bladder distension due to ANS involvement                         (1)
Case No 30
A 2 years boy was unconscious for 1 min following head injury.

Although he is fully alert now there is bruising on the left side
  of the head over the parietal bone.

Skull x-rays are performed and he is admitted for neurological
  observations.
Questions
1.   What clinical features would suggest requirement
     of CT skull and/ or neurosurgical opinion?
2.   What non-surgical temporary measures can be
     used to reduce raised intracranial pressure?
3.   What advice needs to be given to parents if after 24
     h the child has remained well and is ready for
     discharge?
Answers
1.        Neurosurgical Opinion/ CT skull
     a.     Deteriorating level of consciousness
     b.     Focal neurological signs
     c.     Depressed Skull fracture
     d.     Basal Skull fracture
     e.     Seizures
     f.     CSF leak
     g.     Signs of Raised ICT                    (2)
Answers
2.    Temporary manoeuvres to reduce raised ICT
      Nursing in a 30 degrees head up position
      Diuretics (Mannitol)
      Hypertonic saline
      Artificial hyperventilation
                                                  (2)
Answers
3.   At discharge it should be stressed that child should
     be brought back if he develops any of following:
      Vomiting
      Drowsy/ altered sensorium
      Blurred/ double vision
      Seizures                                     (2)
CASE SCENARIO
 7 year old girl presents with high irregular fever
 for 3 weeks, each episode associated with
 erythematous rashes over the trunk and
 proximal extremities, pain and swelling of Rt
 knee and easy fatigability. Has leucocytosis,
 raised ESR, normal chest skiagram and
 negative Mx.
1.   Give most probable diagnosis.
2.   List other expected systemic clinical findings
3.   Give value of serology in diagnosis
4.   Outline steps in management
5.   List prognostic factors
1. JRA – Systemic                                   0.5
2. Systemic exam findings
   a. Hepato – splenomegaly                         0.5
   b. Lymphadenopathy                               0.5
   c. Serous cavity effusions                       0.5
3. Serology
   a. ANA    -     Negative                         0.5
   b. RF     -     Negative                         0.5
4. Management
   a. “Pyramid” approach – NSAIDS, chloroquine,   1.0
      methotrexate,azothiaprine/cyclophosphamide.
   b. Steroids                                    0.5
   c. TNF-alpha receptor antagonists              0.5
5. Number and severity of joint involvement 1.0
                                (Total Marks 6.0)
This patient becomes ill with thrombocytopenia,
profound anemia, and markedly elevated transaminases
probably has what complication?
Macrophage activation syndrome
CASE Scenario
       12 yr old boy, case of Haemophilia A, is
       admitted with spontaneous haemarthrosis.

  1.     What is the level of factor VIII ?
  2.     Factor VIII in treatment.
        a.   Level of factor VIII to be achieved
        b.   Dose of factor VIII
  3.     The patient has to undergo major surgery
        a.   Level of factor VIII to be achieved
        b.   Dose of factor VIII
  4. What are the other precautions?
  5. What is the role of DDAVP in haemophilia A ?
1.        <1%                                                           0.5
2.        Factor VIII in treatment
     a.     35-40%                                                      1.0
     b.     20 units/kg, repeat daily if required                       1.0
            till joint normalises
3.        Dose in major surgery
     a.     100%                                                        1.0
     b.     50 units/kg, infuse 2-3 units/kg/hr to                      1.0
            maintain level at 100%for 24 hrs
            and then >50% for 7 days
4.        Precautions
     a.     Avoid Anti-inflammatory drugs with anti-platelet            0.5
            action
     b.     Screen for Transfusion transmitted diseases if              0.5
            plasma products are used
4.        Desmopressin Acetate: Release of factor VIII in mild
          cases                                              0.5
                                                        (Total marks 6.0)
CASE Scenario
 A 7 yr old girl with strong family history of
 chronic fatal liver disease presents with features of
 haemolysis.
  1. List one clinical examination which may give the
     diagnosis
  2.State most probable group of aetiology based on answer
     to question ‘1’
  3.List three most important tests for diagnosis with
     values
  4.Outline the management of this child
1.     Kayser-Fleischer rings                          0.5
2.     Wilson disease                                  0.5
3.     Diagnostic tests
     a.Serum ceruloplasmin                             0.5
        < 200 mg/lit                                   0.5
     b. Urinary copper                                 0.5
        >100 microgram/day                             0.5
     c. Hepatic copper                                 0.5
        >250 microgram/gm of dry weight                0.5
1.     Management
     a. Restrict copper intake                         0.5
     b.   Chelation with penicillamine                 0.5
     c.   B6 supplementation                           0.5
     d.   Liver transplant
                  0.5
                                          (Total marks 6.0)
Drugs & Devices
Identify the instrument.
Which illness is it used in ?
What are the three settings ?
Peak Flow Meter
Bronchial Asthma

1. Green zone >80% Peak Exp Flow
2. Yellow Zone 50-80 % PEF
3. Red zone < 50% PEF




                                   Total 3 marks
Which class does this
  antibiotic belong to ?
What is the spectrum?
What is the dosage?
What is the imp clinical
  use?
4 marks
Oxazolidinones
Gram-positive organisms including MRSA
Penicillin-resistant pneumococci
Gram-negative anaerobes
Mycobacteria
10mg/kg/dose 8 hrly
Multiresistant gram-positive organisms, including
  MRSA who have failed to respond to
 vancomycin.
Indications for use of this
  compound?
what is the dose?
What are the indications
When should it be started?
What are the side effects of
  parenteral nutrition?
4 marks
Parental nutrition
Dose 0.5 -3.5 g/kg/day
When enteral nutrition is insufficient or contra-
 indicated.
Day 1
Dyselectronemia, cholestasis, bacterial or fungal
 colonisation and sepsis, thrombosis, azotemia,
 thrombocytopenia
Identify the
 product?
What are the
 advantages of its
 use?
Enumerate five
 complications?
PICC                                                              1
Preserves veins, allows concentrated infusates, permits long term IV
  alimentation
       1
6 complications-                                                  3
Catheter fracture and embolisation,
Leakage
 Thrombosis of catheter
Accidental displacement / Tip Migration
Deep Venous Thrombosis
Infection
What is the equipment ?

Identify the parts.
Enumerate indications for its
 use.
What are the contra-
 indications?
How do you sterilise it?

5 marks
Self inflating 250 ml ambu bag                               1
Oxygen , air inlet , pt outlet, valve assembly, oxygen reservoir
 and pressure release valve.                                  1

Gasping/apneic infant, cyanosis inspite of free flow oxygen and
 HR<100/m 30secs after initial steps.
       1
MSAF and suspected cong diaph hernia                      1
 Wash with detergent and hot water. Dry thoroughly. If visibly
 soiled with blood/body fluids, clean and send to Central
 Sterilization Dept                                        1
Identify the product?
What is the dosage?
What are the indications for use?

Enumerate three precautions ?
 Surfactant                                            0.5
 4ml/kg                                                0.5
 Surfactant replacement therapy for prevention and treatment of RDS
  in preterms,                  0.5

 Survanta should not be removed from the refrigerator for more than
  24 hours and it should not be warmed and returned to the refrigerator
  more than once.
 Do not suction the infant for 1 hour after dosing unless signs of
  significant airway obstruction occur.
 Marked improvements in oxygenation and decrease in lung
  compliance may occur within minutes of administration of
  SURVANTA. Therefore, frequent and careful clinical observation and
  monitoring.
 Donot shake the vial                             1.5
What is the equipment?
What are the indications of its
 use?
What are the advantages?
Double lumen endotracheal tube                1

For surfactant instillation                   1
 To aid lung lavage

Permits efficient drug instillation without
 interrupting ventilation                          1
Identify the equipment
Indications for usage
Complications (mention any three)
 Bubble CPAP apparatus                                  1
 Indications -                                          2
    •   If FiO2 requirement is > 0.3
    •   SA Score > 4
    •   ABG Score > 3 on oxygen therapy
    •   Frequent apnea with documented hypoxemia
    •   Clinically significant chest retractions after extubation from
        mechanical ventilation
• Complications                                          1
        •   Air leak
        •   Reduced venous return
        •   IVH
Which class does this
 antibiotic belong to ?
What is the spectrum?
What is the dosage?
What is the two imp side
 effects?
Carbapenems
Widest spectrum of antimicrobial activity among the
  β-lactam antibiotics against both gm-ve and gm +
  organisms except MRSA.
20 - 40mg/kg/dose 12hrly
Transient agranulocytosis, thrombocytopenia.
Seizures



                                           2 marks
ABG1
pH 7.28

PaCO254mmHg

PaO2 45mmHg

HCO3 29mEq/L

Base excess    +7
What are the-

Abnormalities?   1

Diagnosis?       1

Likely causes?   1
Low pH, high PaCO2, low PaO2, high HCO3




Uncompensated respiratory acidosis with hypoxemia


RESPIRATORY FAILURE (pneumonia, RDS)
ABG 2
pH            7.57
PaCO2         22mmHg
PaO2          156mmHg
HCO3          18mEq/L
Base excess   -8
What are the-
Abnormalities?   1
Diagnosis?       1
Likely cause?    1
High pH, low PaCO2, high PaO2, low HCO3



UNCOMPENSATED RESPIRATORY ALKALOSIS
 WITH HYPEROXIA

Hyperventilation with high FiO2
ABG 3
   pH            7.32
   PaCO2         30mmHg

   PaO2          70mmHg

   HCO3          12mEq/L
   Base excess   -8
QUESTIONS
   ABNORMALITIES     1
   DIAGNOSIS         1

   LIKELY CAUSES      1
• Low pH, low PaCO2, normal PaO2, low HCO3
• UNCOMPENSATED METABOLIC ACIDOSIS
• SHOCK, RENAL FAILURE
ABG 4
   pH            7.60
   PaCO2         21mmHg

   PaO2          65mmHg

   HCO3          24mEq/L
   Base excess   +2
QUESTIONS
   ABNORMALITIES        1

   DIAGNOSIS            1

   LIKELY CAUSES        1
High pH, low PaCO2, normal PaO2, normal HCO3
UNCOMPENSATED RESPIRATORY ALKALOSIS


HYPERVENTILATION (BRONCHIAL ASTHMA)
ABG 5
   pH            7.36
   PaCO2         70mmHg

   PaO2          75mmHg

   HCO3          35mEq/L
   Base excess   +14
QUESTIONS
   ABNORMALITIES   1

   DIAGNOSIS       1

   LIKELY CAUSES   1
Low pH, high PaCO2, normal PaO2, high HCO3


COMPENSATED RESPIRATORY ACIDOSIS



VENTILATED INFANT WITH TUBE BLOCK
ABG 6
   pH            7.52
   PaCO2         47mmHg

   PaO2          80mmHg

   HCO3          30mEq/L
   Base excess   +3
QUESTIONS

High pH, high PaCO2, normal PaO2, high HCO3


UNCOMPENSATED METABOLIC ALKALOSIS



VOMITING, PYLORIC STENOSIS
ABG 7
   pH            7.14
   PaCO2         54mmHg

   PaO2          55mmHg

   HCO3          14 mEq/L
   Base excess   -7
QUESTIONS
   ABNORMALITIES     1
   DIAGNOSIS        1
   LIKELY CAUSES   1
   Low pH, high PaCO2, normal PaO2, low HCO3



   UNCOMPENSATED RESPIRATORY AND METABOLIC
    ACIDOSIS

   INFANT ON VENTILATOR WITH TUBE BLOCK &
    SHOCK
Describe the
 lesion.
What is the DD ?
Brain Stem cell glioma,




                           1 mark
Observed Station 1
DRUG COUNSELLING

4 yr old boy weighing 15 Kg , diagnosed as Nephrotic
 Syndrome to be started on steroids.
1.   Introduces self/ puts the child
     and attendant at ease.                                        0.5
2.   Explains the disease in simple words      0.5
3.   Explains the medication
        Dose                                                      0.5
        Frequency                                                 0.5
        Relationship with meals                                   0.5
4.   Explains side effects
        GIT                                                       0.5
        Steroid effect                                            0.5
5.   Explains monitoring of response
        Urinary output                                            0.5
        Body weight                                               0.5
6.   Asks for queries if any.                                      0.5
7.   Advises to report back if any problems.           0.5
8.   Advises about pain abdomen                                    0.5
                                               (Total marks 6.0)
Observed Station 2
DRUG COUNSELING


A 8yr old boy, weighing 30 kg, diagnosed as DM Type
 I requires 30 units insulin/day,
 in two divided doses ,combining short and
 intermediate acting insulins.
 Counsel regarding administration.
1.Introduces self & puts child and      0.5
                                                                                     attendant at ease.
                                                                                  2. Explains the problem in brief.     0.5
                                                                                  3. Familiarizes the attendant with the injectables     0.5
                                                                                     and the syringes.
                                                                                  4. Explains dose, time of injection and relationship   2.0
                                                                                     with meals, sites, route and rotation of sites
                                                                                  5. Explains the calculation of dose in ml.     1.0
                                                                                  6. Explains the sequence of loading (regular first).   0.5
                                                                                  7. Briefs about hypoglycemia incl mgt. 0.5
                                                                                  8. Inquires about any doubts and advises to report     0.5 back in
                                                          case of any problems.
    (Dose – 2/3rd before breakfast,1/3rd before dinner)
    (Ratio intermediate /regular 2:1/3:1)
                                                                                     (Total marks 6.0)
Observed Station 3
COUNSELING

24 yr old lady diagnosed as HIV + at 36 weeks of gestation.
  Counsel regarding perinatal transmission and follow up.
1.    Ensures the presence of husband                                             0.5
2.    Introduces self/ puts the couple at ease.                       0.5
3.    Explains the disease in simple words                            0.5
4.    Explains the incidence and modes of perinatal
       transmission
          20-30%                                                      0.5
          Prenatal                                                    0.5
          Intranatal                                                              0.5
          Breast feeding                                                          0.5
5.    Explains modalities of reducing rate of transmission
          ART to mother and child                                     1.0
          LSCS Vs Vaginal delivery                                    1.0
          Breast feeding Vs top feeding                                           1.0
6.    Explains effect of measures – reduction by             50%                  1.0
7.    Explains screening of the infant                                0.5
8.    Explains safety of cuddling, petting and kissing                1.0
9.    Asks for queries if any.                                        0.5
10.   Advises to report back if any problems.                         0.5

                                                             (Total marks 10.0)
Observed Station 4
Drug Counseling
4 yr old child (15Kg) ,a case of gen tonic-clonic
 seizures has been advised syr carbamezapine (100
 mg/5ml).Counsel the mother regarding
 administration.
1.    Introduces self & puts child and           0.5
      attendant at ease.
2.    Explains the problem in brief.                   0.5
3.    Familiarizes the attendant with the drug         0.5
      and dispenser
4.    Explains dose(75 mg-3.5 ml), in 3 divided 0.5 doses
5.    Advises not to miss the dose                     0.5
6.    Increase to (150 mg – 7.5 ml)                    0.5
7.    Explains side effects                        1.0
8.    Calls for blood counts weekly                0.5
9.    Calls for serum levels of the drug           1.0
10.   Advises to report in case of doubt               0.5
                                 (Total marks 6.0)
Observed Station 5
DRUG COUNSELING


2 yr old child weighing 10 Kg is being discharged after
  admission for febrile seizure. Counsel the mother regarding
  domiciliary management of seizure with rectal diazepam.
1.    Introduces self & puts child and                        0.5
      attendant at ease.
2.    Explains the problem in brief.                          0.5
3.    Familiarizes the attendant with the drug,               0.5
       the syringe and the catheter.
4.    Explains dose(3mg), and loading of the syringe          0.5
5.    Explains the position of the child (lateral)            0.5
6.    Explains introduction of the catheter (lubrication,     1.0
      and length of introduction)
7.    Explains pushing the drug                                     0.5
8.    Advises pinching of the buttocks together after         1.0
9.    Withdrawing the catheter)
10.   Advises one repetition after 15 mts                           0.5
11.   Explains side effects                                 0.5

                                       (Total marks 6.0)
http://groups.yahoo.com/group/PediatricsDNB/

                   oscepediatrics.blogspot.in/

Más contenido relacionado

La actualidad más candente

La actualidad más candente (20)

OSCE in Pediatrics (Wadia, Sept 2011)
OSCE in Pediatrics (Wadia, Sept 2011)OSCE in Pediatrics (Wadia, Sept 2011)
OSCE in Pediatrics (Wadia, Sept 2011)
 
Mock OSCE Pediatrics Apr 2013
Mock OSCE Pediatrics Apr 2013Mock OSCE Pediatrics Apr 2013
Mock OSCE Pediatrics Apr 2013
 
OSCE - Pune mock OSCE 2012
OSCE - Pune mock OSCE 2012OSCE - Pune mock OSCE 2012
OSCE - Pune mock OSCE 2012
 
OSCE Pediatrics KKCTH
OSCE Pediatrics KKCTHOSCE Pediatrics KKCTH
OSCE Pediatrics KKCTH
 
Pediatrics OSCE
Pediatrics OSCEPediatrics OSCE
Pediatrics OSCE
 
Mock OSCE in Pediatrics Apr 2014 Part 1 qn ans
Mock OSCE in Pediatrics Apr 2014 Part 1 qn ansMock OSCE in Pediatrics Apr 2014 Part 1 qn ans
Mock OSCE in Pediatrics Apr 2014 Part 1 qn ans
 
OSCE Pediatrics
OSCE PediatricsOSCE Pediatrics
OSCE Pediatrics
 
OSCE MAY 2022-PART-4 -PAED.pptx
OSCE MAY 2022-PART-4 -PAED.pptxOSCE MAY 2022-PART-4 -PAED.pptx
OSCE MAY 2022-PART-4 -PAED.pptx
 
DNB OSCE SGRH - 2
DNB OSCE SGRH - 2DNB OSCE SGRH - 2
DNB OSCE SGRH - 2
 
Pediatric Neurology OSCE (PG CME -Wadia)
Pediatric Neurology OSCE (PG CME -Wadia)Pediatric Neurology OSCE (PG CME -Wadia)
Pediatric Neurology OSCE (PG CME -Wadia)
 
Rainbow Hospital OSCE
Rainbow Hospital OSCERainbow Hospital OSCE
Rainbow Hospital OSCE
 
Paediatrics quiz
Paediatrics quizPaediatrics quiz
Paediatrics quiz
 
Pediatrics OSCE pictures
Pediatrics OSCE pictures Pediatrics OSCE pictures
Pediatrics OSCE pictures
 
Ospe peads
Ospe peadsOspe peads
Ospe peads
 
Pediatrics OSCE Observed Stations Oct 2013
Pediatrics OSCE Observed Stations Oct 2013Pediatrics OSCE Observed Stations Oct 2013
Pediatrics OSCE Observed Stations Oct 2013
 
DNB Pediatrics OSCE June 2013
DNB Pediatrics OSCE June 2013DNB Pediatrics OSCE June 2013
DNB Pediatrics OSCE June 2013
 
OSCE BBH 2011
OSCE BBH 2011OSCE BBH 2011
OSCE BBH 2011
 
genetic osce for dnb pdiatric
genetic osce for dnb pdiatricgenetic osce for dnb pdiatric
genetic osce for dnb pdiatric
 
Practical pediatric quiz - Kaun Banega Winner
Practical pediatric quiz - Kaun Banega WinnerPractical pediatric quiz - Kaun Banega Winner
Practical pediatric quiz - Kaun Banega Winner
 
Infectious diseases
Infectious diseasesInfectious diseases
Infectious diseases
 

Destacado

How to Save 33 Years of Your Life Using Habits
How to Save 33 Years of Your Life Using HabitsHow to Save 33 Years of Your Life Using Habits
How to Save 33 Years of Your Life Using HabitsImran Esmail
 
Ppt emotional i ntelligence
Ppt emotional i ntelligencePpt emotional i ntelligence
Ppt emotional i ntelligencesmritibhambra
 
The New Multiscreen World By Google
The New Multiscreen World By GoogleThe New Multiscreen World By Google
The New Multiscreen World By Googleservicesmobiles.fr
 
Assembly language 8086
Assembly language 8086Assembly language 8086
Assembly language 8086John Cutajar
 
Journalising- easy way to learn journal entries for beginners in Accounting S...
Journalising- easy way to learn journal entries for beginners in Accounting S...Journalising- easy way to learn journal entries for beginners in Accounting S...
Journalising- easy way to learn journal entries for beginners in Accounting S...Sarat Kumar Budumuru
 
General pathology lecture 5 inflammation & repair
General pathology lecture 5 inflammation & repairGeneral pathology lecture 5 inflammation & repair
General pathology lecture 5 inflammation & repairLheanne Tesoro
 
OSCE Pediatrics Observed Stations (Mock Exam Apr 2013)
OSCE Pediatrics Observed Stations (Mock Exam Apr 2013)OSCE Pediatrics Observed Stations (Mock Exam Apr 2013)
OSCE Pediatrics Observed Stations (Mock Exam Apr 2013)Dr Padmesh Vadakepat
 
Enfermedades producidas por bacterias, virus, hongos
Enfermedades producidas por bacterias, virus, hongos Enfermedades producidas por bacterias, virus, hongos
Enfermedades producidas por bacterias, virus, hongos Beatriz Isabel Robles Rios
 
Corporate Governance
Corporate GovernanceCorporate Governance
Corporate GovernanceAliza Racelis
 
Work Hacks : Body Language
Work Hacks : Body LanguageWork Hacks : Body Language
Work Hacks : Body LanguageInterQuest Group
 
Discrete Mathematics - All chapters
Discrete Mathematics - All chapters Discrete Mathematics - All chapters
Discrete Mathematics - All chapters Omnia A. Abdullah
 
Pakistan Education Plan
Pakistan Education PlanPakistan Education Plan
Pakistan Education PlanMinhaaj Rehman
 
Top Office Etiquette Mistakes
Top Office Etiquette MistakesTop Office Etiquette Mistakes
Top Office Etiquette Mistakesej4video
 
Size exclusion chromatography
Size exclusion chromatographySize exclusion chromatography
Size exclusion chromatographyMandvi Shandilya
 
Payment and Settlement Systems(SWIFT,NEFT and Securities Cycle)
Payment and Settlement Systems(SWIFT,NEFT and Securities Cycle)Payment and Settlement Systems(SWIFT,NEFT and Securities Cycle)
Payment and Settlement Systems(SWIFT,NEFT and Securities Cycle)Savita Marwal
 

Destacado (19)

How to Save 33 Years of Your Life Using Habits
How to Save 33 Years of Your Life Using HabitsHow to Save 33 Years of Your Life Using Habits
How to Save 33 Years of Your Life Using Habits
 
Ppt emotional i ntelligence
Ppt emotional i ntelligencePpt emotional i ntelligence
Ppt emotional i ntelligence
 
The New Multiscreen World By Google
The New Multiscreen World By GoogleThe New Multiscreen World By Google
The New Multiscreen World By Google
 
Assembly language 8086
Assembly language 8086Assembly language 8086
Assembly language 8086
 
Journalising- easy way to learn journal entries for beginners in Accounting S...
Journalising- easy way to learn journal entries for beginners in Accounting S...Journalising- easy way to learn journal entries for beginners in Accounting S...
Journalising- easy way to learn journal entries for beginners in Accounting S...
 
Bacteriology
BacteriologyBacteriology
Bacteriology
 
General pathology lecture 5 inflammation & repair
General pathology lecture 5 inflammation & repairGeneral pathology lecture 5 inflammation & repair
General pathology lecture 5 inflammation & repair
 
OSCE Pediatrics Observed Stations (Mock Exam Apr 2013)
OSCE Pediatrics Observed Stations (Mock Exam Apr 2013)OSCE Pediatrics Observed Stations (Mock Exam Apr 2013)
OSCE Pediatrics Observed Stations (Mock Exam Apr 2013)
 
Enfermedades producidas por bacterias, virus, hongos
Enfermedades producidas por bacterias, virus, hongos Enfermedades producidas por bacterias, virus, hongos
Enfermedades producidas por bacterias, virus, hongos
 
Corporate Governance
Corporate GovernanceCorporate Governance
Corporate Governance
 
Work Hacks : Body Language
Work Hacks : Body LanguageWork Hacks : Body Language
Work Hacks : Body Language
 
Lean Manufacturing - Toyota Production System
Lean Manufacturing - Toyota Production SystemLean Manufacturing - Toyota Production System
Lean Manufacturing - Toyota Production System
 
Discrete Mathematics - All chapters
Discrete Mathematics - All chapters Discrete Mathematics - All chapters
Discrete Mathematics - All chapters
 
Mineral & energy resources
Mineral & energy resourcesMineral & energy resources
Mineral & energy resources
 
Pakistan Education Plan
Pakistan Education PlanPakistan Education Plan
Pakistan Education Plan
 
Types of Irrigation
Types of IrrigationTypes of Irrigation
Types of Irrigation
 
Top Office Etiquette Mistakes
Top Office Etiquette MistakesTop Office Etiquette Mistakes
Top Office Etiquette Mistakes
 
Size exclusion chromatography
Size exclusion chromatographySize exclusion chromatography
Size exclusion chromatography
 
Payment and Settlement Systems(SWIFT,NEFT and Securities Cycle)
Payment and Settlement Systems(SWIFT,NEFT and Securities Cycle)Payment and Settlement Systems(SWIFT,NEFT and Securities Cycle)
Payment and Settlement Systems(SWIFT,NEFT and Securities Cycle)
 

Similar a DNB Pediatrics OSCE CME (Command Hospital, Pune)

Fournier's gangrene
Fournier's gangreneFournier's gangrene
Fournier's gangrenesbcoomes
 
Benign Thyroid-Y6.ppt
Benign Thyroid-Y6.pptBenign Thyroid-Y6.ppt
Benign Thyroid-Y6.pptEmanZayed17
 
Vasculitis Overview
Vasculitis OverviewVasculitis Overview
Vasculitis Overviewjcm MD
 
Testicular Cancer
Testicular CancerTesticular Cancer
Testicular CancerRyan Mills
 
Traditional medicine 2007 by ap u kyaw naing
Traditional medicine 2007 by ap u kyaw naingTraditional medicine 2007 by ap u kyaw naing
Traditional medicine 2007 by ap u kyaw naingyinnshang
 
Role of spleenectomy in itp
Role of spleenectomy in itpRole of spleenectomy in itp
Role of spleenectomy in itpSajid Ali
 
Aipgmee2011part1
Aipgmee2011part1Aipgmee2011part1
Aipgmee2011part1isha9
 
Congenital heart disease
Congenital heart diseaseCongenital heart disease
Congenital heart diseasesurendra sharma
 
Mod 1 case 2022
Mod 1 case 2022Mod 1 case 2022
Mod 1 case 2022vetindex4
 
(FULL) CME Glass (of milk) Half Full copy.pdf
(FULL) CME Glass (of milk) Half Full copy.pdf(FULL) CME Glass (of milk) Half Full copy.pdf
(FULL) CME Glass (of milk) Half Full copy.pdfgdbnqj7b9v
 
Ospe for mbbs imagings
Ospe for mbbs   imagingsOspe for mbbs   imagings
Ospe for mbbs imagingsRajat Biswas
 

Similar a DNB Pediatrics OSCE CME (Command Hospital, Pune) (20)

Osce bat surgery
Osce bat surgeryOsce bat surgery
Osce bat surgery
 
Fournier's gangrene
Fournier's gangreneFournier's gangrene
Fournier's gangrene
 
Hydatid cyst disease
Hydatid cyst diseaseHydatid cyst disease
Hydatid cyst disease
 
Pulmonary Sarcoidosis
Pulmonary SarcoidosisPulmonary Sarcoidosis
Pulmonary Sarcoidosis
 
Benign Thyroid-Y6.ppt
Benign Thyroid-Y6.pptBenign Thyroid-Y6.ppt
Benign Thyroid-Y6.ppt
 
Vasculitis Overview
Vasculitis OverviewVasculitis Overview
Vasculitis Overview
 
Testicular Cancer
Testicular CancerTesticular Cancer
Testicular Cancer
 
Traditional medicine 2007 by ap u kyaw naing
Traditional medicine 2007 by ap u kyaw naingTraditional medicine 2007 by ap u kyaw naing
Traditional medicine 2007 by ap u kyaw naing
 
EMERGENCY RED FLAGS
EMERGENCY RED FLAGSEMERGENCY RED FLAGS
EMERGENCY RED FLAGS
 
Sarcoidosis
SarcoidosisSarcoidosis
Sarcoidosis
 
Medicine ospe
Medicine ospeMedicine ospe
Medicine ospe
 
Role of spleenectomy in itp
Role of spleenectomy in itpRole of spleenectomy in itp
Role of spleenectomy in itp
 
Aipgmee2011part1
Aipgmee2011part1Aipgmee2011part1
Aipgmee2011part1
 
Moeez
Moeez Moeez
Moeez
 
Os i2
Os i2Os i2
Os i2
 
Pitu
PituPitu
Pitu
 
Congenital heart disease
Congenital heart diseaseCongenital heart disease
Congenital heart disease
 
Mod 1 case 2022
Mod 1 case 2022Mod 1 case 2022
Mod 1 case 2022
 
(FULL) CME Glass (of milk) Half Full copy.pdf
(FULL) CME Glass (of milk) Half Full copy.pdf(FULL) CME Glass (of milk) Half Full copy.pdf
(FULL) CME Glass (of milk) Half Full copy.pdf
 
Ospe for mbbs imagings
Ospe for mbbs   imagingsOspe for mbbs   imagings
Ospe for mbbs imagings
 

Más de Dr Padmesh Vadakepat

Neonatal Nursing of Extremely Premature Neonates - Dr Padmesh
Neonatal Nursing of Extremely Premature Neonates - Dr PadmeshNeonatal Nursing of Extremely Premature Neonates - Dr Padmesh
Neonatal Nursing of Extremely Premature Neonates - Dr PadmeshDr Padmesh Vadakepat
 
Update on Antenatal Steroids 2021 - Dr Padmesh
Update on Antenatal Steroids 2021  - Dr PadmeshUpdate on Antenatal Steroids 2021  - Dr Padmesh
Update on Antenatal Steroids 2021 - Dr PadmeshDr Padmesh Vadakepat
 
Inhaled Nitric Oxide (iNO) in Newborns - Dr Padmesh - Neonatology
Inhaled Nitric Oxide (iNO) in Newborns - Dr Padmesh - NeonatologyInhaled Nitric Oxide (iNO) in Newborns - Dr Padmesh - Neonatology
Inhaled Nitric Oxide (iNO) in Newborns - Dr Padmesh - NeonatologyDr Padmesh Vadakepat
 
Approach to Ano Rectal Malformations - Dr Padmesh - Neonatology
Approach to Ano Rectal Malformations - Dr Padmesh - NeonatologyApproach to Ano Rectal Malformations - Dr Padmesh - Neonatology
Approach to Ano Rectal Malformations - Dr Padmesh - NeonatologyDr Padmesh Vadakepat
 
Blood Group Selection in Newborn Transfusion - Dr Padmesh - Neonatology
Blood Group Selection in Newborn Transfusion  - Dr Padmesh - NeonatologyBlood Group Selection in Newborn Transfusion  - Dr Padmesh - Neonatology
Blood Group Selection in Newborn Transfusion - Dr Padmesh - NeonatologyDr Padmesh Vadakepat
 
Vaccination in Preterms by - Dr Padmesh - Neonatology
Vaccination in Preterms by  - Dr Padmesh - NeonatologyVaccination in Preterms by  - Dr Padmesh - Neonatology
Vaccination in Preterms by - Dr Padmesh - NeonatologyDr Padmesh Vadakepat
 
European Consensus Statement on RDS 2019
European Consensus Statement on RDS 2019European Consensus Statement on RDS 2019
European Consensus Statement on RDS 2019Dr Padmesh Vadakepat
 
Blood Brain Barrier by Dr Padmesh V
Blood Brain Barrier by Dr Padmesh VBlood Brain Barrier by Dr Padmesh V
Blood Brain Barrier by Dr Padmesh VDr Padmesh Vadakepat
 
Humidication in NICU - Dr Padmesh - Neonatology
Humidication in NICU - Dr Padmesh - NeonatologyHumidication in NICU - Dr Padmesh - Neonatology
Humidication in NICU - Dr Padmesh - NeonatologyDr Padmesh Vadakepat
 
Subgaleal Hemorrhage - Dr Padmesh - Neonatology
Subgaleal Hemorrhage - Dr Padmesh - NeonatologySubgaleal Hemorrhage - Dr Padmesh - Neonatology
Subgaleal Hemorrhage - Dr Padmesh - NeonatologyDr Padmesh Vadakepat
 
Touch and Massage Therapy in Newborn - Dr Padmesh V
Touch and Massage Therapy in Newborn - Dr Padmesh VTouch and Massage Therapy in Newborn - Dr Padmesh V
Touch and Massage Therapy in Newborn - Dr Padmesh VDr Padmesh Vadakepat
 
Perinatal infections- Diagnosis & Management - Dr Padmesh - Neonatology
Perinatal infections- Diagnosis & Management  - Dr Padmesh - NeonatologyPerinatal infections- Diagnosis & Management  - Dr Padmesh - Neonatology
Perinatal infections- Diagnosis & Management - Dr Padmesh - NeonatologyDr Padmesh Vadakepat
 
Shock & Inotropes in Neonates - Dr Padmesh - Neonatology
Shock & Inotropes in Neonates  - Dr Padmesh - NeonatologyShock & Inotropes in Neonates  - Dr Padmesh - Neonatology
Shock & Inotropes in Neonates - Dr Padmesh - NeonatologyDr Padmesh Vadakepat
 
Assessment of Fetal Well being - Dr Padmesh - Neonatology
Assessment of Fetal Well being - Dr Padmesh - NeonatologyAssessment of Fetal Well being - Dr Padmesh - Neonatology
Assessment of Fetal Well being - Dr Padmesh - NeonatologyDr Padmesh Vadakepat
 
Ballard score.. - Dr Padmesh - Neonatology
Ballard score..  - Dr Padmesh - NeonatologyBallard score..  - Dr Padmesh - Neonatology
Ballard score.. - Dr Padmesh - NeonatologyDr Padmesh Vadakepat
 
European Consensus Guidelines- RDS in Preterm Newborns
European Consensus Guidelines- RDS in Preterm NewbornsEuropean Consensus Guidelines- RDS in Preterm Newborns
European Consensus Guidelines- RDS in Preterm NewbornsDr Padmesh Vadakepat
 
Pulmonary Abscess in Children .. Dr Padmesh
Pulmonary Abscess in Children .. Dr PadmeshPulmonary Abscess in Children .. Dr Padmesh
Pulmonary Abscess in Children .. Dr PadmeshDr Padmesh Vadakepat
 

Más de Dr Padmesh Vadakepat (20)

Neonatal Nursing of Extremely Premature Neonates - Dr Padmesh
Neonatal Nursing of Extremely Premature Neonates - Dr PadmeshNeonatal Nursing of Extremely Premature Neonates - Dr Padmesh
Neonatal Nursing of Extremely Premature Neonates - Dr Padmesh
 
Update on Antenatal Steroids 2021 - Dr Padmesh
Update on Antenatal Steroids 2021  - Dr PadmeshUpdate on Antenatal Steroids 2021  - Dr Padmesh
Update on Antenatal Steroids 2021 - Dr Padmesh
 
Inhaled Nitric Oxide (iNO) in Newborns - Dr Padmesh - Neonatology
Inhaled Nitric Oxide (iNO) in Newborns - Dr Padmesh - NeonatologyInhaled Nitric Oxide (iNO) in Newborns - Dr Padmesh - Neonatology
Inhaled Nitric Oxide (iNO) in Newborns - Dr Padmesh - Neonatology
 
Approach to Ano Rectal Malformations - Dr Padmesh - Neonatology
Approach to Ano Rectal Malformations - Dr Padmesh - NeonatologyApproach to Ano Rectal Malformations - Dr Padmesh - Neonatology
Approach to Ano Rectal Malformations - Dr Padmesh - Neonatology
 
ROP - Dr Padmesh - Neonatology
ROP  - Dr Padmesh - NeonatologyROP  - Dr Padmesh - Neonatology
ROP - Dr Padmesh - Neonatology
 
Blood Group Selection in Newborn Transfusion - Dr Padmesh - Neonatology
Blood Group Selection in Newborn Transfusion  - Dr Padmesh - NeonatologyBlood Group Selection in Newborn Transfusion  - Dr Padmesh - Neonatology
Blood Group Selection in Newborn Transfusion - Dr Padmesh - Neonatology
 
Vaccination in Preterms by - Dr Padmesh - Neonatology
Vaccination in Preterms by  - Dr Padmesh - NeonatologyVaccination in Preterms by  - Dr Padmesh - Neonatology
Vaccination in Preterms by - Dr Padmesh - Neonatology
 
European Consensus Statement on RDS 2019
European Consensus Statement on RDS 2019European Consensus Statement on RDS 2019
European Consensus Statement on RDS 2019
 
Blood Brain Barrier by Dr Padmesh V
Blood Brain Barrier by Dr Padmesh VBlood Brain Barrier by Dr Padmesh V
Blood Brain Barrier by Dr Padmesh V
 
Humidication in NICU - Dr Padmesh - Neonatology
Humidication in NICU - Dr Padmesh - NeonatologyHumidication in NICU - Dr Padmesh - Neonatology
Humidication in NICU - Dr Padmesh - Neonatology
 
Subgaleal Hemorrhage - Dr Padmesh - Neonatology
Subgaleal Hemorrhage - Dr Padmesh - NeonatologySubgaleal Hemorrhage - Dr Padmesh - Neonatology
Subgaleal Hemorrhage - Dr Padmesh - Neonatology
 
Touch and Massage Therapy in Newborn - Dr Padmesh V
Touch and Massage Therapy in Newborn - Dr Padmesh VTouch and Massage Therapy in Newborn - Dr Padmesh V
Touch and Massage Therapy in Newborn - Dr Padmesh V
 
Perinatal infections- Diagnosis & Management - Dr Padmesh - Neonatology
Perinatal infections- Diagnosis & Management  - Dr Padmesh - NeonatologyPerinatal infections- Diagnosis & Management  - Dr Padmesh - Neonatology
Perinatal infections- Diagnosis & Management - Dr Padmesh - Neonatology
 
Shock & Inotropes in Neonates - Dr Padmesh - Neonatology
Shock & Inotropes in Neonates  - Dr Padmesh - NeonatologyShock & Inotropes in Neonates  - Dr Padmesh - Neonatology
Shock & Inotropes in Neonates - Dr Padmesh - Neonatology
 
ABC of ABG - Dr Padmesh
ABC of ABG - Dr PadmeshABC of ABG - Dr Padmesh
ABC of ABG - Dr Padmesh
 
Assessment of Fetal Well being - Dr Padmesh - Neonatology
Assessment of Fetal Well being - Dr Padmesh - NeonatologyAssessment of Fetal Well being - Dr Padmesh - Neonatology
Assessment of Fetal Well being - Dr Padmesh - Neonatology
 
Ballard score.. - Dr Padmesh - Neonatology
Ballard score..  - Dr Padmesh - NeonatologyBallard score..  - Dr Padmesh - Neonatology
Ballard score.. - Dr Padmesh - Neonatology
 
European Consensus Guidelines- RDS in Preterm Newborns
European Consensus Guidelines- RDS in Preterm NewbornsEuropean Consensus Guidelines- RDS in Preterm Newborns
European Consensus Guidelines- RDS in Preterm Newborns
 
Say NO to drugs .. Dr.Padmesh
Say NO to drugs .. Dr.PadmeshSay NO to drugs .. Dr.Padmesh
Say NO to drugs .. Dr.Padmesh
 
Pulmonary Abscess in Children .. Dr Padmesh
Pulmonary Abscess in Children .. Dr PadmeshPulmonary Abscess in Children .. Dr Padmesh
Pulmonary Abscess in Children .. Dr Padmesh
 

Último

Cardiac Output, Venous Return, and Their Regulation
Cardiac Output, Venous Return, and Their RegulationCardiac Output, Venous Return, and Their Regulation
Cardiac Output, Venous Return, and Their RegulationMedicoseAcademics
 
Bhawanipatna Call Girls 📞9332606886 Call Girls in Bhawanipatna Escorts servic...
Bhawanipatna Call Girls 📞9332606886 Call Girls in Bhawanipatna Escorts servic...Bhawanipatna Call Girls 📞9332606886 Call Girls in Bhawanipatna Escorts servic...
Bhawanipatna Call Girls 📞9332606886 Call Girls in Bhawanipatna Escorts servic...Dipal Arora
 
Circulatory Shock, types and stages, compensatory mechanisms
Circulatory Shock, types and stages, compensatory mechanismsCirculatory Shock, types and stages, compensatory mechanisms
Circulatory Shock, types and stages, compensatory mechanismsMedicoseAcademics
 
Chandigarh Call Girls Service ❤️🍑 9809698092 👄🫦Independent Escort Service Cha...
Chandigarh Call Girls Service ❤️🍑 9809698092 👄🫦Independent Escort Service Cha...Chandigarh Call Girls Service ❤️🍑 9809698092 👄🫦Independent Escort Service Cha...
Chandigarh Call Girls Service ❤️🍑 9809698092 👄🫦Independent Escort Service Cha...Sheetaleventcompany
 
Call Girls Rishikesh Just Call 9667172968 Top Class Call Girl Service Available
Call Girls Rishikesh Just Call 9667172968 Top Class Call Girl Service AvailableCall Girls Rishikesh Just Call 9667172968 Top Class Call Girl Service Available
Call Girls Rishikesh Just Call 9667172968 Top Class Call Girl Service Availableperfect solution
 
Difference Between Skeletal Smooth and Cardiac Muscles
Difference Between Skeletal Smooth and Cardiac MusclesDifference Between Skeletal Smooth and Cardiac Muscles
Difference Between Skeletal Smooth and Cardiac MusclesMedicoseAcademics
 
Call Girls in Lucknow Just Call 👉👉8630512678 Top Class Call Girl Service Avai...
Call Girls in Lucknow Just Call 👉👉8630512678 Top Class Call Girl Service Avai...Call Girls in Lucknow Just Call 👉👉8630512678 Top Class Call Girl Service Avai...
Call Girls in Lucknow Just Call 👉👉8630512678 Top Class Call Girl Service Avai...soniyagrag336
 
Independent Bangalore Call Girls (Adult Only) 💯Call Us 🔝 7304373326 🔝 💃 Escor...
Independent Bangalore Call Girls (Adult Only) 💯Call Us 🔝 7304373326 🔝 💃 Escor...Independent Bangalore Call Girls (Adult Only) 💯Call Us 🔝 7304373326 🔝 💃 Escor...
Independent Bangalore Call Girls (Adult Only) 💯Call Us 🔝 7304373326 🔝 💃 Escor...Sheetaleventcompany
 
Kolkata Call Girls Shobhabazar 💯Call Us 🔝 8005736733 🔝 💃 Top Class Call Gir...
Kolkata Call Girls Shobhabazar  💯Call Us 🔝 8005736733 🔝 💃  Top Class Call Gir...Kolkata Call Girls Shobhabazar  💯Call Us 🔝 8005736733 🔝 💃  Top Class Call Gir...
Kolkata Call Girls Shobhabazar 💯Call Us 🔝 8005736733 🔝 💃 Top Class Call Gir...Namrata Singh
 
Race Course Road } Book Call Girls in Bangalore | Whatsapp No 6378878445 VIP ...
Race Course Road } Book Call Girls in Bangalore | Whatsapp No 6378878445 VIP ...Race Course Road } Book Call Girls in Bangalore | Whatsapp No 6378878445 VIP ...
Race Course Road } Book Call Girls in Bangalore | Whatsapp No 6378878445 VIP ...dishamehta3332
 
Exclusive Call Girls Bangalore {7304373326} ❤️VVIP POOJA Call Girls in Bangal...
Exclusive Call Girls Bangalore {7304373326} ❤️VVIP POOJA Call Girls in Bangal...Exclusive Call Girls Bangalore {7304373326} ❤️VVIP POOJA Call Girls in Bangal...
Exclusive Call Girls Bangalore {7304373326} ❤️VVIP POOJA Call Girls in Bangal...Sheetaleventcompany
 
Pune Call Girl Service 📞9xx000xx09📞Just Call Divya📲 Call Girl In Pune No💰Adva...
Pune Call Girl Service 📞9xx000xx09📞Just Call Divya📲 Call Girl In Pune No💰Adva...Pune Call Girl Service 📞9xx000xx09📞Just Call Divya📲 Call Girl In Pune No💰Adva...
Pune Call Girl Service 📞9xx000xx09📞Just Call Divya📲 Call Girl In Pune No💰Adva...Sheetaleventcompany
 
Ahmedabad Call Girls Book Now 8980367676 Top Class Ahmedabad Escort Service A...
Ahmedabad Call Girls Book Now 8980367676 Top Class Ahmedabad Escort Service A...Ahmedabad Call Girls Book Now 8980367676 Top Class Ahmedabad Escort Service A...
Ahmedabad Call Girls Book Now 8980367676 Top Class Ahmedabad Escort Service A...Genuine Call Girls
 
Call Girls Mussoorie Just Call 8854095900 Top Class Call Girl Service Available
Call Girls Mussoorie Just Call 8854095900 Top Class Call Girl Service AvailableCall Girls Mussoorie Just Call 8854095900 Top Class Call Girl Service Available
Call Girls Mussoorie Just Call 8854095900 Top Class Call Girl Service AvailableJanvi Singh
 
👉 Chennai Sexy Aunty’s WhatsApp Number 👉📞 7427069034 👉📞 Just📲 Call Ruhi Colle...
👉 Chennai Sexy Aunty’s WhatsApp Number 👉📞 7427069034 👉📞 Just📲 Call Ruhi Colle...👉 Chennai Sexy Aunty’s WhatsApp Number 👉📞 7427069034 👉📞 Just📲 Call Ruhi Colle...
👉 Chennai Sexy Aunty’s WhatsApp Number 👉📞 7427069034 👉📞 Just📲 Call Ruhi Colle...rajnisinghkjn
 
Dehradun Call Girls Service {8854095900} ❤️VVIP ROCKY Call Girl in Dehradun U...
Dehradun Call Girls Service {8854095900} ❤️VVIP ROCKY Call Girl in Dehradun U...Dehradun Call Girls Service {8854095900} ❤️VVIP ROCKY Call Girl in Dehradun U...
Dehradun Call Girls Service {8854095900} ❤️VVIP ROCKY Call Girl in Dehradun U...Sheetaleventcompany
 
Call girls Service Phullen / 9332606886 Genuine Call girls with real Photos a...
Call girls Service Phullen / 9332606886 Genuine Call girls with real Photos a...Call girls Service Phullen / 9332606886 Genuine Call girls with real Photos a...
Call girls Service Phullen / 9332606886 Genuine Call girls with real Photos a...call girls hydrabad
 
Cara Menggugurkan Kandungan Dengan Cepat Selesai Dalam 24 Jam Secara Alami Bu...
Cara Menggugurkan Kandungan Dengan Cepat Selesai Dalam 24 Jam Secara Alami Bu...Cara Menggugurkan Kandungan Dengan Cepat Selesai Dalam 24 Jam Secara Alami Bu...
Cara Menggugurkan Kandungan Dengan Cepat Selesai Dalam 24 Jam Secara Alami Bu...Cara Menggugurkan Kandungan 087776558899
 
Low Cost Call Girls Bangalore {9179660964} ❤️VVIP NISHA Call Girls in Bangalo...
Low Cost Call Girls Bangalore {9179660964} ❤️VVIP NISHA Call Girls in Bangalo...Low Cost Call Girls Bangalore {9179660964} ❤️VVIP NISHA Call Girls in Bangalo...
Low Cost Call Girls Bangalore {9179660964} ❤️VVIP NISHA Call Girls in Bangalo...Sheetaleventcompany
 
💚Chandigarh Call Girls Service 💯Piya 📲🔝8868886958🔝Call Girls In Chandigarh No...
💚Chandigarh Call Girls Service 💯Piya 📲🔝8868886958🔝Call Girls In Chandigarh No...💚Chandigarh Call Girls Service 💯Piya 📲🔝8868886958🔝Call Girls In Chandigarh No...
💚Chandigarh Call Girls Service 💯Piya 📲🔝8868886958🔝Call Girls In Chandigarh No...Sheetaleventcompany
 

Último (20)

Cardiac Output, Venous Return, and Their Regulation
Cardiac Output, Venous Return, and Their RegulationCardiac Output, Venous Return, and Their Regulation
Cardiac Output, Venous Return, and Their Regulation
 
Bhawanipatna Call Girls 📞9332606886 Call Girls in Bhawanipatna Escorts servic...
Bhawanipatna Call Girls 📞9332606886 Call Girls in Bhawanipatna Escorts servic...Bhawanipatna Call Girls 📞9332606886 Call Girls in Bhawanipatna Escorts servic...
Bhawanipatna Call Girls 📞9332606886 Call Girls in Bhawanipatna Escorts servic...
 
Circulatory Shock, types and stages, compensatory mechanisms
Circulatory Shock, types and stages, compensatory mechanismsCirculatory Shock, types and stages, compensatory mechanisms
Circulatory Shock, types and stages, compensatory mechanisms
 
Chandigarh Call Girls Service ❤️🍑 9809698092 👄🫦Independent Escort Service Cha...
Chandigarh Call Girls Service ❤️🍑 9809698092 👄🫦Independent Escort Service Cha...Chandigarh Call Girls Service ❤️🍑 9809698092 👄🫦Independent Escort Service Cha...
Chandigarh Call Girls Service ❤️🍑 9809698092 👄🫦Independent Escort Service Cha...
 
Call Girls Rishikesh Just Call 9667172968 Top Class Call Girl Service Available
Call Girls Rishikesh Just Call 9667172968 Top Class Call Girl Service AvailableCall Girls Rishikesh Just Call 9667172968 Top Class Call Girl Service Available
Call Girls Rishikesh Just Call 9667172968 Top Class Call Girl Service Available
 
Difference Between Skeletal Smooth and Cardiac Muscles
Difference Between Skeletal Smooth and Cardiac MusclesDifference Between Skeletal Smooth and Cardiac Muscles
Difference Between Skeletal Smooth and Cardiac Muscles
 
Call Girls in Lucknow Just Call 👉👉8630512678 Top Class Call Girl Service Avai...
Call Girls in Lucknow Just Call 👉👉8630512678 Top Class Call Girl Service Avai...Call Girls in Lucknow Just Call 👉👉8630512678 Top Class Call Girl Service Avai...
Call Girls in Lucknow Just Call 👉👉8630512678 Top Class Call Girl Service Avai...
 
Independent Bangalore Call Girls (Adult Only) 💯Call Us 🔝 7304373326 🔝 💃 Escor...
Independent Bangalore Call Girls (Adult Only) 💯Call Us 🔝 7304373326 🔝 💃 Escor...Independent Bangalore Call Girls (Adult Only) 💯Call Us 🔝 7304373326 🔝 💃 Escor...
Independent Bangalore Call Girls (Adult Only) 💯Call Us 🔝 7304373326 🔝 💃 Escor...
 
Kolkata Call Girls Shobhabazar 💯Call Us 🔝 8005736733 🔝 💃 Top Class Call Gir...
Kolkata Call Girls Shobhabazar  💯Call Us 🔝 8005736733 🔝 💃  Top Class Call Gir...Kolkata Call Girls Shobhabazar  💯Call Us 🔝 8005736733 🔝 💃  Top Class Call Gir...
Kolkata Call Girls Shobhabazar 💯Call Us 🔝 8005736733 🔝 💃 Top Class Call Gir...
 
Race Course Road } Book Call Girls in Bangalore | Whatsapp No 6378878445 VIP ...
Race Course Road } Book Call Girls in Bangalore | Whatsapp No 6378878445 VIP ...Race Course Road } Book Call Girls in Bangalore | Whatsapp No 6378878445 VIP ...
Race Course Road } Book Call Girls in Bangalore | Whatsapp No 6378878445 VIP ...
 
Exclusive Call Girls Bangalore {7304373326} ❤️VVIP POOJA Call Girls in Bangal...
Exclusive Call Girls Bangalore {7304373326} ❤️VVIP POOJA Call Girls in Bangal...Exclusive Call Girls Bangalore {7304373326} ❤️VVIP POOJA Call Girls in Bangal...
Exclusive Call Girls Bangalore {7304373326} ❤️VVIP POOJA Call Girls in Bangal...
 
Pune Call Girl Service 📞9xx000xx09📞Just Call Divya📲 Call Girl In Pune No💰Adva...
Pune Call Girl Service 📞9xx000xx09📞Just Call Divya📲 Call Girl In Pune No💰Adva...Pune Call Girl Service 📞9xx000xx09📞Just Call Divya📲 Call Girl In Pune No💰Adva...
Pune Call Girl Service 📞9xx000xx09📞Just Call Divya📲 Call Girl In Pune No💰Adva...
 
Ahmedabad Call Girls Book Now 8980367676 Top Class Ahmedabad Escort Service A...
Ahmedabad Call Girls Book Now 8980367676 Top Class Ahmedabad Escort Service A...Ahmedabad Call Girls Book Now 8980367676 Top Class Ahmedabad Escort Service A...
Ahmedabad Call Girls Book Now 8980367676 Top Class Ahmedabad Escort Service A...
 
Call Girls Mussoorie Just Call 8854095900 Top Class Call Girl Service Available
Call Girls Mussoorie Just Call 8854095900 Top Class Call Girl Service AvailableCall Girls Mussoorie Just Call 8854095900 Top Class Call Girl Service Available
Call Girls Mussoorie Just Call 8854095900 Top Class Call Girl Service Available
 
👉 Chennai Sexy Aunty’s WhatsApp Number 👉📞 7427069034 👉📞 Just📲 Call Ruhi Colle...
👉 Chennai Sexy Aunty’s WhatsApp Number 👉📞 7427069034 👉📞 Just📲 Call Ruhi Colle...👉 Chennai Sexy Aunty’s WhatsApp Number 👉📞 7427069034 👉📞 Just📲 Call Ruhi Colle...
👉 Chennai Sexy Aunty’s WhatsApp Number 👉📞 7427069034 👉📞 Just📲 Call Ruhi Colle...
 
Dehradun Call Girls Service {8854095900} ❤️VVIP ROCKY Call Girl in Dehradun U...
Dehradun Call Girls Service {8854095900} ❤️VVIP ROCKY Call Girl in Dehradun U...Dehradun Call Girls Service {8854095900} ❤️VVIP ROCKY Call Girl in Dehradun U...
Dehradun Call Girls Service {8854095900} ❤️VVIP ROCKY Call Girl in Dehradun U...
 
Call girls Service Phullen / 9332606886 Genuine Call girls with real Photos a...
Call girls Service Phullen / 9332606886 Genuine Call girls with real Photos a...Call girls Service Phullen / 9332606886 Genuine Call girls with real Photos a...
Call girls Service Phullen / 9332606886 Genuine Call girls with real Photos a...
 
Cara Menggugurkan Kandungan Dengan Cepat Selesai Dalam 24 Jam Secara Alami Bu...
Cara Menggugurkan Kandungan Dengan Cepat Selesai Dalam 24 Jam Secara Alami Bu...Cara Menggugurkan Kandungan Dengan Cepat Selesai Dalam 24 Jam Secara Alami Bu...
Cara Menggugurkan Kandungan Dengan Cepat Selesai Dalam 24 Jam Secara Alami Bu...
 
Low Cost Call Girls Bangalore {9179660964} ❤️VVIP NISHA Call Girls in Bangalo...
Low Cost Call Girls Bangalore {9179660964} ❤️VVIP NISHA Call Girls in Bangalo...Low Cost Call Girls Bangalore {9179660964} ❤️VVIP NISHA Call Girls in Bangalo...
Low Cost Call Girls Bangalore {9179660964} ❤️VVIP NISHA Call Girls in Bangalo...
 
💚Chandigarh Call Girls Service 💯Piya 📲🔝8868886958🔝Call Girls In Chandigarh No...
💚Chandigarh Call Girls Service 💯Piya 📲🔝8868886958🔝Call Girls In Chandigarh No...💚Chandigarh Call Girls Service 💯Piya 📲🔝8868886958🔝Call Girls In Chandigarh No...
💚Chandigarh Call Girls Service 💯Piya 📲🔝8868886958🔝Call Girls In Chandigarh No...
 

DNB Pediatrics OSCE CME (Command Hospital, Pune)

  • 1. COMMAND HOSPITAL PUNE 25TH July 2008
  • 2. INTRODUCTION to Objective structured clinical examination Observation Stations & Clinical Examination 5 min each
  • 3. OBSERVED STATION OBSERVED STATION OBSERVED STATION OBSERVED REST STATION STATION REST STATION STATION 1 OBSERVED REST STATION STATION
  • 4. G cen AB ario G Kar y Bio otype Blank OSCE Sheets sta tisti cs
  • 5.
  • 6.
  • 7.
  • 8.
  • 9.
  • 10.
  • 11. 1. STATIONS: 2. OBSERVED STATION: Foreign Body Apiration: demonstrate action on a manikin 3. Statistics 4. Neonatal Chest X ray with MAS and umbilical venous cannulation. Qs on complications and heparin infusion 5. Rest Station 6. Spirometry 7. OBSERVED STATION: History Taking: examiner is observing your procedures. 8. History taking of a 4 year old child with febrile seizures—have all the points to diagnose, prognosticate and manage the child covered. 9. Congenital Anomaly: Qs on diagnosis, prognosis and causes of death. 10. Photomicrograph of a PBS of a haemolytic anaemia. Qs on the abnormalities of the RBCs, the diagnosis, two inv, two treatment stratigies. 11. X- Ray Chest of a CHD. Qs on radiological findings, diagnosis, antenatal history, DD, definitive manangement of the condition. 12. Cavernous sinus thrombosis 13. OBSERVED STATION: drug counselling to the mother of a child with bronchial asthma: from the drugs to devices. 14. Photomicrograph of a PBS of different types of mononuclear WBCs. Qs on identification of the cells. 15. Photograph of a dermatological condition. Qs on diagnosis, etiology, management. 16. Vaccine: Qs on a particular vaccine, its storage, route, indications. 17. Drug: Qs on the mechanism of action, group, metaboism, SEs. 18. OBSERVED STATION: Examination of a system. 19. Photograph of a Karyotype. Qs on the abnormality, diagnosis, 3 CFs, type of inheritance. 20. Micronutrients and their deficiency disorders. 21. Biostatics: calculation of relative risk, Odd’s ratio and Attributable risk. 22. Photograph of an ectoparasite. Qs on identification, disease caused, drug therapy. 23. ECG: abnormality, axis, diagnosis, complications, management. 24. OBSERVED STATION: examiner asses the skills. Newborn gestational age assessment. 25. REST STATION 26. Neonatal: HMD:Qs four risk factors, ventilation modalitiws, ABG: a/A ratio, A-aDO2, OI for the ABG & ventilator settings. 27. Lab Test: Haemolytic anaemia, antenatal diagnosis, management. 28. Short Stature scenario with Ht, Wt, SMR, Bone age given. Qs on endocrine inv. 29. Drugs: doses, routes, antidotes. 30. Clinical Situation: GBS 31. Photograph of oxygen delivery devices: Qs on Fio2 rates delivered. 32.  REST STATION 
  • 13. If patient presents with fever and toxaemia, give three differential diagnosis  Give three modalities of management
  • 14. 1. Lung Abcess 2. Infected Bronchogenic cyst 3. Infected Hydatid Cyst 1. Antibiotics (anaerobic +aerobic) 2. Chest physiotherapy 3. Percutaneous CT guided aspiration 0.5 each Total 3 marks
  • 15. What is the lesion ? Give two management modalities. What complication can occur during management & how can it be prevented?
  • 16. NCC 1 NCC: Anticonvulsants 1 Albendazole Increase in Cerebral Edema 1 Use steroid cover Total 3 marks
  • 17.
  • 18. Describe the lesion. What are the management modalities of such lesions especially if they are leading to complications ?
  • 19. Haemangioma scalp 1 Conservative 2 Oral steroids Local steroids PDL: pulse dyed laser IFN alfa interferon (in case of haemangiomas causing pressure effects or erosion leading to significant morbidity) Total 3 marks
  • 20. Worrisome Hemangiomas Multiple cutaneous hemangiomas: Associated with visceral hemangiomas (e.g., liver) Large hemangiomas: May cause significant disfigurement of underlying structures and may be associated with congestive heart failure "Beard" hemangiomas: May be a marker for underlying laryngeal or subglottic hemangioma that may impair respiratory function Midline spinal hemangiomas: May be a marker for underlying spinal cord abnormality Head and neck hemangiomas: May be associated with other congenital anomalies, including central nervous system, cardiac, ocular, and sternal defects (e.g., Posterior fossa malformation, Hemangioma, Arterial abnormalities, Coarctation, Eye abnormalities, Sternal defects [PHACES] syndrome). Vulnerable anatomic locations: Impair vital functions, cause disfigurement (e.g. periocular, neck, lip, nasal tip) Ulcerated hemangiomas: Increased risk of superinfection, cause pain and lead to scarring
  • 21. What are two imp radiological abnormalities? What is the diagnosis ? What are the three imp investigations ? What are the metabolic abnormalities expected? What is the management ?
  • 22.  Distension of stomach  No gas in the intestines  Pyloric Stenosis  USS abdomen: pyloric thickness >4mm/ pyloric length >14 mm  S electrolytes  Ba studies  Hypochloremic, hypokalemic, metabolic alkalosis  Management of the fluid & electrolytes  Ramstedt’s pyloromyotomy 0.5 each Total 5 marks
  • 23. What is the diagnosis ? What is the likely organism ?
  • 24. Pnematoceles 1 Likely organism Staphylococcus 1 Total 2 mark
  • 25. Give five radiological findings ? What is the diagnosis ? What is the management ?
  • 26. 1 Ground glass appearance of bone 2 2 Thinned cortex 3 Periosteal calcification 4 White line of Fraenkel (well calcified cartilage) 5 Wimberger’s sign (white ring) Scurvy 1 1 Vit C 100-200 mg/ day1 2 Dietary Therapy Total 4 marks
  • 27. What was the lesion ? What was the procedure carried out ? What are the complications ?
  • 28. VSD 1 Device implant 1 Device displacement 2 Emboli formation Haemolytic Anaemia Total 3 marks
  • 29. Name the Investigation ? What is the diagnosis ? What are the further investigations ?
  • 30. MCU Posterior urethral Valves DTPA (diethylene tetra amine pentoic acid) Function DMSA (dimercapto succinic acid) -scarring Total 3 marks
  • 31.
  • 32. What is the abnormality? What is the diagnosis ? How is it suspected clinically ? What is the management ?
  • 33. 1. .Bowelloops in lt hemithorax, mediastinal shift to right 1 2. Congenital diaphragmatic hernia 1 3. Respiratory distress Mediastinal shift Bowel sounds in the thorax Scaphoid abdomen 2 4. Avoid B&M vent prop up and decompress stomach Ventilate Treat PPHN Surgical correction Antenatal tracheal ligation 3
  • 34. CDH Pleuroperitoneal membrane defect Fetal lung development Pseudoglandular stage 5 to 16 weeks Canalicular 16 to 24 weeks Saccular stage Alveolar stage continues till 8 years of life
  • 35. What is the abnormality? What is the likely diagnosis? Delineate management. Mention 3 complications
  • 36. Multiple fluid levels. gasless lower abdomen 1 Small bowel obstruction 1 Surgical correction 1 Dyselectrolytemia 1 Perforation Exaggerated hyperbilirubinemia Dysmotility syndrome Martin: Fanaroff and Martin's Neonatal-Perinatal Medicine, 8th ed.,
  • 37.
  • 38. Describe the X –ray Diagnosis Give three recent advances in management
  • 39. Hyper inflated lung fields with areas of atelectasis interspersed with areas of overinflation 1 MAS 1 Lung lavage surfactant instillation HFOV NO liquid ventilation 2
  • 40.
  • 41. What is the abnormality Three high risk situations when this condition is imminent Management
  • 42. Pneumothorax 1 CDH Ventilation Post surfactant ventilation 2 Chest tube drainage 1
  • 43.
  • 44. What is the abnormality What is it a complication of
  • 45. Pneumomediastinum 1 Forceful ventilation 1
  • 46.
  • 47. Identify the abnormality Clinical presentation Treatment
  • 48. Pneumopericardium 1 Shock with weak pulses 1 Drainage 1
  • 49. What is the diagnosis ?
  • 51. What are the MRI findings ? What is the DD What further inv will you do on the CSF to confirm the diagnosis ?
  • 53.
  • 54. Oligoclonal bands are also found in:  Multiple sclerosis  Devic's disease  Systemic lupus erythematosus  Neurosarcoidosis  SSPE  Subarachnoid haemorrhage  Syphilis  CNS Lymphoma
  • 55. What are the MRI findings ? What could be the etiology ?
  • 56. Multifocal cerebral & cerebellar encephalomalacia Sequel of ischaemic injury in the perinatal period. 2 marks
  • 57. What is the likely diagnosis ? What is the IP? Which is the commonest site?
  • 58. TB Spine 1 2 YRS 1 Thoraclumbar area of max stress valveless venous drainage 2 4 mark
  • 59. What are the MRI findings ? Likely etiology Likely CSF picture
  • 60. TBM Increased proteins 3 mark Increased cells likely lymphocytes Low sugar
  • 61. What is the X ray suggestive of ? What is the likely organism? What are the complications? What is the drug of choice? What is the duration of therapy?
  • 62. Lobar Pneumonia Pnemococous/Staphylococcus Synpneumonic effusion, empyema, Penicillin for susceptible org and cefotaxime / vancomycin for penicillin resistant org 10-14 d 5 marks
  • 63.
  • 64. Questions 1. What is the anatomical structure in which coin is lodged? 2. What is the location of carina with respect to thoracic vertebrae? 3. What are the anatomical areas of esophageal narrowing? 4. How can this foreign body be removed?
  • 65. Answers 1 Esophagus When foreign bodies lodge in the esophagus, the flat surface of the object is seen in the AP view * T 4 1 Anatomic areas of esophageal narrowing 3 Cricoid Tracheal bifurcation Gastro-esophageal junction Endoscopy 1 *
  • 66. L
  • 67. Question 1. What is the diagnosis? 2. Fill in the blanks In an exudative pleural fluid a. Proteins > b. Pleural Fluid LDH > …………. c. Fluid to serum LDH ratio > ……….. d. Cell count ……… 3. What does VATS stand for?
  • 68. Answers 2 1. Pleural Effusion (Right) 2. Fill in the blanks In an exudative pleural fluid 0.5 a. Proteins > 3.0 g/dL 0.5 b. Pleural Fluid LDH > 200 IU/L c. Fluid to serum LDH ratio > 0.6 0.5 d. Cell count > 1000 0.5 3. Video Assisted Thoracoscopic Surgery 1
  • 69.
  • 70. Questions 1. Diagnosis? 2. What are the first two steps in treatment of hypoxic spell? 3. In a cyanotic newborn, how can you distinguish pulmonary disease from cyanotic congenital heart disease? 4. Which cardiac conditions are associated with following a. Egg Shaped Heart b. Snowman silhouette c. Rib notching
  • 71. Answers 1.5 1. Cyanotic Congenital Heart Disese Probably TOF  The heart size is normal  Pulmonary vascular markings are decreased  A hypoplastic main pulmonary artery segment contributes to the formation of the “boot-shaped” heart. Pediatric cardiology Myung K Park 5th ed
  • 72. Answers 1 2. Knee Chest Position 1 Morphine 1 3. Hyperoxia Test 4. X-ray appearances a. Egg Shaped Heart Transposition of great arteries 0.5 b. Snowman silhouette Total anomalous pulmonary venous return (supracardiac) 0.5 c. Rib notching Co-arctation of aorta (long standing) 0.5 Park: Pediatric Cardiology for Practitioners, 5th ed.
  • 73. 3 day neonate with Lethargy Feed refusal Abdominal distension
  • 74. Questions 1. What stage of NEC is depicted in the X-ray? 2. What is the radiological feature of Bell stage III NEC? 3. Name two more conditions associated with pneumatosis intestinalis?
  • 75. Answers 1 1. NEC Stage II 1 2. Pneumoperitoneum 2 3. Any two of following Hirschsprung's disease, Pseudomembranous enterocolitis, Neonatal ulcerative colitis, Ischemic bowel disease PART 5 Neonatal Necrotizing Enterocolitis Martin: Fanaroff and Martin's Neonatal-Perinatal Medicine, 8th ed.,
  • 76. 6 weeks infant Case of Cholestatic jaundice (Extra-Hepatic Biliary Atresia) c/o swelling left thigh
  • 77. Questions 1. What is the likely cause of fracture femur in this case? 2. How can this complication be prevented? 3. How do you manage pruritus in these patients? 4. An infant with cholestasis, triangular facies, and a pulmonic stenosis murmur is likely to have what syndrome?
  • 78. Answers 1 1. Metabolic Bone disease (secondary to Vitamin D deficiency due to malabsorption of fat soluble vitamins) 1 2. Replace 5,000-8,000 U /d of D2,or 3 -5 µg/kq/d of 25-hydroxycholecalciferol 3. Ursodeoxycholic acid 15-20 mg/kg/day 1 4. Alagille syndrome 1 (Arteriohepatic dysplasia)
  • 79.
  • 80. L Previously healthy 7 years girl c/o c Sudden onset weakness right upper and lower limb t Facial palsy right (UMN) Normal sensorium No fever/ trauma/ seizures
  • 81. Questions 1. What is the level of lesion on MRI? 2. What are the structures marked c t 3. Which hemoglobinopathy can be associated with this kind of presentation? 4. A dilated and unreactive pupil indicates the compression of what structure?
  • 82. Answers 2 1. Infarct in the left basal ganglia, the posterior limb of internal capsule, and the head of the caudate 1
  • 83. Answers 2. C Caudate 0.5 nucleus T Thalamus 0.5 P Putamen G Globus pallidus White arrows indicate the ant and post limbs of internal capsule 1
  • 84. Answers 1 1 3. Sickle cell anemia 1 4. Compression of 3rd cranial nerve 1
  • 85.
  • 86. Questions? 1. Diagnosis? 2. What is the treatment (mention complete schedule)?
  • 87. Answers 1 1. Miliary tuberculosis 2. 2HRZE + 7HR 1 IAP Group 4
  • 88.
  • 89. 1. Diagnosis 2. What are the embryologic events that lead to this development? 3. What are three causes of respiratory distress in a baby born with this condition?
  • 90. Answers 1 1. Congenital Diagphragmatic Hernia 2. The posterolateral portion of the diaphragm has remained 1 open between the ninth and tenth weeks of gestation as a result the viscera will pass into the chest, and a CDH will result. 3. a) Mechanical compression of the lungs from the herniated 1 viscera b) Pulmonary hypoplasia from compression of the developing lungs in utero 1 c) Pulmonary hypertension 1
  • 91.
  • 92. Questions Describe the lesion? Give two D/D What is the triad of tumor lysis syndrome?
  • 93. Answers 1 Osteolytic lesion of skull 2 Histiocytosis Ewing`s Sarcoma Lymphoma 3 Bone cyst Hyperuricemia, hyperkalemia, and hyperphosphatemia
  • 94.
  • 95. Questions 1. Describe the X-ray appearance 2. Pathogenesis of the appearance 3. Possible Diagnosis 4. Which disorder is most commonly associated with an elevated MCHC? 5. How is the corrected reticulocyte count calculated?
  • 96. Answers 1. Sunray appearance 1 2. Medullary widening 1 3. Chronic hemolytic anaemia 1 4. Hereditary spherocytosis 1 5. Corrected retic count = reticulocyte % × (patient Hct/normal Hct) 1
  • 97. Below is a midline sagittal cut of a MRI scan of the brain. View the midline anatomic diagram of the brain and identify the following structures
  • 98. Answer S - Suprasellar cistern P0 - Pons P - Midbrain (cerebral peduncles) M - Medulla C - Quadrigeminal plate (superior and inferior colliculi) Q - Quadrigeminal cistern V - Fourth ventricle 1 mark each
  • 99.
  • 100. Question 1. Diagnosis? 2. What is the emergency management of the condition? 3. What is subsequent management after the emergency management is over?
  • 101. Answers 1 1. Pneumothorax, 1 with mediastinal shift 1 2. Put in a needle in second intercostal space 3. Intercostal drain 1
  • 102.
  • 103. Questions 1. What is the diagnosis? 2. Describe three features seen on the X-ray of the disease? 3. What biochemical test would help clinch the diagnosis? 4. What is the treatment of the condition?
  • 104. Answers 1 1. Rickets 0.5 2. a) Cupping 0.5 b) Widening of distal end of metaphysis 0.5 c) Fraying 3. Calcium, Phosphorus, Alkaline phosphatase 1.5 4. Injection Vitamin D 6 lac unit IM stat PO Calcium 1 1
  • 105.
  • 106. Questions 1. What is the diagnosis? 2. What is the clinical sign for the diagnosis called as? 3. Name one intervention which can lead to this?
  • 107. Answers 1 1. Pneumopericardium 1 2. Hammans sign 1 3. Invasive ventilation with high pressures
  • 108. X-ray neck lateral view in a child with respiratory distress
  • 109. Questions 1. What is the diagnosis? 2. Which is the commonest organism implicated in this disease? 3. What antibiotics are useful in this condition?
  • 110. Answers 1. Epiglottitis 1 2. Hemophilus influenzae type B 1 3. Cephalosporins/ Ampicillin/ sulbactam 1
  • 111.
  • 112. Questions 1. Diagnosis 2. By what gestational age would this defect occur? 3. This can be prevented in subsequent pregnancies by intake of Folic acid. Folic acid should be taken in what dose and started when?
  • 113. Answers 1 1. Occipital Encephalocele 1 2. 26 days post conception 3. Folic acid Dose: 0.4mg/day 1 Periconceptional period 1
  • 114. Questions 1. Diagnosis 2. What is the clinical picture? 3. What is the requirement of echo before surgery?
  • 115. Answers 1. Tracheo-esophageal fistula 2. Excessive drooling Respiratory distress 3. To rule out associated Congenital heart diseases Right sided aorta PART 3 Selected Thoracic Gastrointestinal Anomalies Martin: Fanaroff and Martin's Neonatal-Perinatal Medicine, 8th ed.,
  • 117. What is the diagnosis ? Name three complications. What is the period of infectivity.
  • 118. • Chicken pox 1 1. Pneumonia 1 2. Encephalitis 3. Ataxia 4. ADEM 5. Progressive varicella: organ invol, h’ge • Airborne or direct contact : contagious 2 days before & till all lesions are dry 1 Total 3 marks
  • 119.
  • 120. 1. Give the diagnosis. 2. Give four important components. 3. State the most important prognostic factors.
  • 121. 1. Prune- Belly Syndrome 0.5 2. Important components a. Abnormal abdominal musculature 0.5 b. Abdominal cryptorchidism0.5 c. Renal and ureteric anomalies with VUR 0.5 d. Pulmonary hypoplasia 0.5 3. Long term complication a. ESRD 0.5 4. Prognostic factors a. Degree of renal dysplasia 0.5 b. Pulmonary hypoplasia` 0.5 (Total marks 4.0)
  • 122.
  • 123. 1. Give the diagnosis. 2. State mode of inheritance. 3. Name five important components. 4. Name most frequent immunological defect.
  • 124. 1. Ataxia – Telengiectasia 0.5 2. Autosomal recessive 0.5 3. Important components a. Cerebellar ataxia 0.5 b. Oculo-cutaneous telengiectasia 0.5 c. Immuno-deficiency 0.5 d. Sino-pulmonary infections 0.5 e. Lympho - reticular malignancies 0.5 4. Selective absence of Ig A 0.5 (Total marks 4.0) (NO MARKS IF FIRST ANSWER IS WRONG)
  • 125.
  • 127. 1. Seckel Syndrome 2.0 2. Important features 2.0 a. Microcephaly b. Beak like nose “Bird face” c. Hypertelorism d. GU anomalies e. Growth retardation (Total marks 4.0) (NO MARKS IF FIRST ANSWER IS WRONG)
  • 128.
  • 129. 1. Give the diagnosis. 2. Give three important complications. 3. List three treatment modalities
  • 130. 1.Giant haemangioma 0.5 2.DIC 1.5
  • 131.
  • 132. 1. State the diagnosis 2. Name the abnormal karyotypes 3. List three prenatal diagnostic techniques 4. Indication for parental screening
  • 133. 1. Down Syndrome 0.5 2. Trisomy 21 1.5 Translocation Mosaicism 3. Triple screen (b-HCG, a fetopr, estradiol)1.5 Ultrasonography Fetal DNA analysis (maternal age >35 yrs) 4. Translocation0.5 (Total marks 4) (No marks if first answer is wrong)
  • 134.
  • 135. What is the condition? 1 Describe the lesions. 1 What is the mode of inheritance? 1 What is CNS association? 1
  • 136. Tuberous sclerosis Ash leaf macule , shagreen patch, angiofibroma Autosomal dominant Characteristic brain lesions are tubers located in convolutions of cerebral hemisphere typically in the subependymal region.
  • 137. What is the condition? How will you treat it?
  • 138. Bitot’s spots Vit A 6 mo to 1 year 1 lac U oral rpt 48 hrs 1 yr to 6 years 2 2 marks
  • 139. What is this condition? Name a metabolic complication arising from this condition 2 marks
  • 141.
  • 142. What is the abnormality seen? What is the diagnosis? What other investigation would you advise in this case?
  • 143. Absence of depression of the right angle of mouth while crying DAOM (deficiency of depressor anguli oris muscle) 2D echocardiography as the condition is associated with cardiac anomalies 3 marks
  • 144. QUESTIONS What is the condition? What is the root value of nerves involved? What is the life threatening complication associated 2 marks
  • 145. Erb- Duchenne palsy  C5,C6 Diaphragmatic paralysis 3 marks Chapter 27 – Birth Injuries Martin: Fanaroff and Martin's Neonatal-Perinatal Medicine, 8th ed
  • 146. Name the condition. What is the natural course of these lesions?
  • 147. Strawberry angiomas Lesions increase over the first few months of life and then regress 2 marks
  • 148. Describe the abnormality 1 What is the likely diagnosis1
  • 149. Webbed neck In a female  Turner syndrome In a male  Noonan’s Syndrome 2 marks
  • 150.
  • 152. Answers Epulis: granular cell tumor of gum Hemangioma Surgical excision  Carbondioxide laser excision 4 marks Chapter 26 – Physical Examination of the Newborn Martin: Fanaroff and Martin's Neonatal-Perinatal Medicine, 8th ed
  • 153.
  • 155. Answers Proptosis ,high forehead,mid facial hypoplasia. Crouzon’s syndrome. Autosomal Dominant 3 marks
  • 156.
  • 157. What is the Diagnosis ? What is the Differential Diagnosis? What are the assoc anomalies? What is the Management ?
  • 158. 1. GASTROCHISIS 1 2. OMPHALOCOELE 1 3. Beckwith Wiedman Trisomies Congenital cardiac anomalies 1 4. Cover defect with sterile draping fluid replacement Early surgical correction 1 PART 4 Selected Abdominal Gastrointestinal Anomalies Martin: Fanaroff and Martin's Neonatal-Perinatal Medicine, 8th ed
  • 159.
  • 160. 1. Give the diagnosis 2. Give five important components 3. Name an important metabolic abnormality and its management
  • 161. 1. Hemihypertrophy Syndrome 0.5 2. Important features a. Macrosomia / (Hemi hypertrophy) 0.5 b. Macroglossia 0.5 c. Hepato-splenomegaly 0.5 d. Omphalocele 0.5 e. Embryonal tumours 0.5 3. Metabolic abnormality & management a. Hyperinsulism Hypoglycaemia 0.5 b. Diazoxide / pancreactectomy 0.5 (Total marks 4.0) Chapter 28 – Congenital Anomalies Martin: Fanaroff and Martin's Neonatal-Perinatal Medicine, 8th ed
  • 162.
  • 163. 1. Give the diagnosis. 2. Name the most important endocrinal feature 3. Name the most important cardiovascular problem 4. Name the mainstay of management
  • 164. 1. Congenital Lipodystrophy 1.0 2. Insulin resistant DM 1.0 3. Hypertrophic cardiomyopathy 1.0 4. Dietary fat regulation 1.0 (Totalmarks 4) (No marks if first answer is wrong)
  • 165.
  • 166. 1. Give the diagnosis 2. List clinical staging. 3. What is the most important sequel and it’s prediction? 4. Outline the treatment.
  • 167. 1. Kawasaki Disease 0.5 2. Clinical Staging a. Acute febrile phase 0.5 b. Sub-acute phase 0.5 c. Convalescent phase 0.5 3. Complication and prediction a. Coronary artery aneurism 0.5 b. Onset of coronary artery changes 0.5 within 2 months of onset of illness 1. Treatment a. IVIG 0.5 b. High dose aspirin 0.5 (Total marks 4.0) (NO MARKS IF FIRST ANSWER IS WRONG)
  • 168.
  • 169. 1. What is the diagnosis? 2. What is the embryologic basis of these defects? 3. In an otherwise healthy child, when are these defects repaired?
  • 170. Answers 1. Bilateral Cleft Lip and Palate 1 2. Cleft lip - Hypoplasia of mesenchymal layer 1 resulting in failure of medial nasal and maxillary process to join Cleft palate - Failure of palatal shelves to fuse 1 3. Cleft lip By 3 Months 1 Cleft palate By one year 1
  • 171. Questions 1. What is the likely diagnosis? 2. What is the mode of inheritance? 3. State True or False a. Limb shortening is greatest in proximal segments. b. Fingers often have a trident configuration. c. Lumbar canal stenosis generally does not develop till early adulthood.
  • 172. Answers 1. Achondroplasia 1 2. Autosomal dominant 1 3. True/ false a. True 1 b. True 1 c. True 1
  • 173.
  • 174.
  • 175. A 3 year girl presents with Severe pruritus esp During night and after Taking bath with warm Water. Pruritus is most Significant in web spaces Of hands and toes Questions 1. Diagnosis 2. What is causative organism? 3. What is Topical Rx? 4. What is oral Rx?
  • 176. Answers 1 1. Scabies 1 2. Sarcoptes scabie 1 3. Permethrin 5% 1 4. Ivermectin
  • 177.  An infant presents with periorificial and acral dermatitis, diarrhea, alopecia and nail dystrophy.
  • 178. Questions 1. Diagnosis? 2. What laboratory test helps to clinch the diagnosis? 3. What is the treatment? 4. Name any three IEM which can have similar finding.
  • 179. Answers 1 1. Acrodermatitis enteropathica 1 2. Low plasma zinc concentration 3. PO Zinc (zinc sulfate, acetate,or gluconate) 50-150mg/day 1 4. Any three of following Maple syrup urine disease, 3 Organic aciduria, Methylmalonic acidemia, Biotinidase deficiency Essential fatty acid deficiency
  • 180. Questions 1. Name the neurocutaneous marker seen 2. In NF-1 what is the diagnostic criteria with respect to this marker? 3. Name a X-linked dominant neurocutaneous syndrome 4. What are the three stages of syndrome in question 3?
  • 181. Answers 1. Café-au lait macule 1 2. Six or more CAL macules larger than 5 mm in greatest diameter in prepubertal And larger than 15 mm in greatest diameter in postpubertal individuals 1 3. Incontinentia pigmenti 4. Stage 1-Vesicular stage Stage 2-Verrucous stage Stage 3-Pigmented stage 1 1 1
  • 182. Questions 1. What is exhibited in the photographs a and b? 2. Diagnosis? a 3. What is the mode of inheritance? 4. What is the major ocular criteria for diagnosis? b
  • 183. Answers 1 1. a: Hypermobility of finger joints. 1 b: Positive thumb (Steinberg) sign 1 2. Marfan syndrome 3. Autosomal dominant 1 4. Ectopia lentis 1 (superior and temporal )
  • 184. 1. Diagnosis 2. What is the pattern of inheritance? 3. What is the characteristic radiographic image called? 4. What is the opthalmologic complication?
  • 185. Answers 1 1. Sturge weber syndrome 1 2. Sporadic 1 3. Tram track appearance 1 4. Bupthalmos
  • 186. Questions 1. What is the diagnosis? 2. What are the three species associated with it? 3. What are useful methods for diagnosing tinea infections?
  • 187. Answers 1 1. Tinea infection 1 2. Trichophyton 1 Microsporum 1 Epidermophyton 3. Potassium hydroxide (KOH) preparations 1 Fungal culture of skin scrapings inoculated on DTM (dermatophyte test media) 1
  • 188.
  • 189. Questions 1. What is the clinical condition called? 2. Which is the commonest malignancy associated with this? 3. Which chromosome is implicated in malignancy mentioned in question 2? 4. What other malignant disorders are associated with the malignancy being discussed?
  • 190. Answers 1 1. Leucocoria 1 2. Retinoblastoma 1 3. RB1 gene Chr 13q 14 1 4. Osteosarcoma Soft tissue sarcomas 1 Malignant melanoma 1
  • 191. Examination Lax skin folds
  • 194. LAB & OTHER INV
  • 195. Lab – 1(Blood) • Age – 4 yrs • Hb – 7 g% • RBCs – Polychromatophilic reticulocytosis, many spherocytes, central pallor less conspicuous • WBC and platelet series- normal • Bone Marrow – Erythroid Hyperplasia • Questions – Differential diagnosis – Confirmatory test
  • 196. Answers: Lab – 1(Blood) • Anemia • Increased retic count – Hemolysis • PBS – spherocytes, other series normal • Bone marrow – hyperplastic – hemolysis • D/D of hemolytic anemia with spherocytes – Hereditary spherocytosis – osmotic fragility – Autoimmune hemolytic anemia – coombs test – Wilsons disease – serum ceruloplasmin – Burns - History Total 2 marks
  • 197. Lab – 2 (Blood) • Age – 3yrs • Hb – 6 gm% • TLC – 1500/cmm • Platelets – 20,000/cmm • Bone marrow – Cellularity reduced • Questions – Diagnosis? – Name 2 infectious causes
  • 198. Answers: Lab – 2 (Blood) All three cell lines are depressed Bone marrow depression Diagnosis- Aplastic anemia Causes CMV Hepatitis B HIV Total 2 marks
  • 199. Lab – 3 (Urine) •Age – 9yrs •Appearance - Turbid •Sp Gr – 1019 •Proteins – 4 + •RBCs – 6 to 10 per HPF, occasional RBC casts seen Questions – Diagnosis? – Causes?
  • 200. Answers: Lab – 3 (Urine) Sp Gr – normal Nephrotic range proteinuria Microscopic hematuria with RBC casts – glomerular pathology Nephrotic syndrome with hematuria – Nephritic onset Nephrotic syndrome Common causes – Idiopathic, SLE, Malaria, HSP, Hepatitis B Total 3 marks
  • 201. Lab – 4 (CSF) Age – 3 days term neonate CSF –  Appearance – clear  Proteins – 10 mg/dl  Glucose – 45 mg/dl ( Blood sugar – 80 mg/dl)  Cells – WBC – 6/cmm pred neutros Questions What is your interpretation
  • 202. Answers: Lab – 4 (CSF) Normal CSF in a neonate WBC 11+/-10 (90th centile-22)  polys 2.2+/-3.8 % (90th centile- 6) Protein 84+/-45 mg/dl Glucose 46+/-10 mg/dl Interpretation – normal CSF report Total 3 marks
  • 203. Lab - 5 Peritoneal fluid • AGE 10 yrs • Peritoneal fluid – Protein 4.5g/dl – LDH 750 IU (90u/L) – WBC 7500/cmm – GLUCOSE 56mg/dl (Blood sugar 80mg/dl) Question – Diagnosis? – Treatment?
  • 204. Answer : Lab - 5 Peritoneal fluid  Normal value Transudate Exudate Sp Gr <1.016 >1.106 Protein <3gm/dl >3gm/dl WBCs <1000/cmm >1000/cmm RBCs <10,000/cmm Variable Glucose = serum <serum pH 7.4 to 7.5 <7.4  Diagnosis Tuberculosis  Treatment- ATT as per IAP consensus Group 4 2 HRZE / 7 HR Total 2 marks
  • 205. 1. What is the diagnosis? 2. What is the mutation that results in this disease? 3. What are the three main categories of crises in patients with sickle cell disease?
  • 206. Answers 1 1. Sickle cell disease 2. On the beta chain, valine is substituted for 1 glutamic acid at position 6 3. Aplastic crisis 1 Vaso-occlusive crisis 1 Acute splenic sequestration 1 Nathan and Oski's Hematology of Infancy and Childhood, 6th ed. 2003, pp 802-811
  • 207. Questions 1. Describe the cells seen. 2. Name one condition each from following category in which these cells are seen a. Neurological b. Metabolic c. Hepatic d. Endocrinal
  • 208. Answers 1 1. Acanthocytosis 2. Conditions 1 a. Neurological Neuroacantocytosis 1 b. Metabolic Abetalipoproteinemia c. Hepatic Severe liver dysfunction 1 d. Endocrinal Hypothyroidism 1
  • 209. What is this modality of investigation? Name three common indications .
  • 210. Answer: Spots: 1 Radionuclide bone scan Indication Metastasis Osteomyelitis Stress fracture Total 2 marks
  • 211. What is this modality of investigation? Name one common indication .
  • 212. Answer : Spots: 2 Hepatobiliary scintigraphy Indication Biliary atresia Total 2 marks
  • 213. What is the diagnosis ? What are the cardiac conditions associated ? What are the endocrinal conditions associated ?
  • 214. Turner’s Syndrome 1. Bicuspid aortic valve 2. Co of aorta 3. AS 4. MVP 5. TAPVR Hypogonadism Hypothyroidism Type-II DM Total 4 marks
  • 215. Questions 1. What is the diagnosis? 2. Give two sexual characters in this condition 3. Give one cardiac condition that may be associated with this condition 4. What would be the plasma levels of a. FSH b. LH c. Testosterone
  • 216. Answers 1 1. Klinefelter Syndrome (47 XXY) 2. Any two 2 Lack secondary sexual characteristics Infertility, azoospermia Gynecomastia Testicular dysgenesis 3. Any one Mitral valve prolapse (55%) 0.5 Varicose veins (20-40%) 4. Plasma levels of a. FSH Increased 0.5 b. LH Increased 0.5 c. Testosterone Decreased 0.5
  • 217. Questions a. Diagnosis b. Commonest differential how do you distinguish between two? c. Give 3 Steps of management
  • 218.
  • 219. Answers PSVT Sinus tachycardia - there is beat to beat variability. Vagal manoeuvres: Carotid Massage Ice packs over face Valsalva manouver Adenosine drug of choice. 4 marks 4 marks
  • 220. ECG
  • 222. Answers ECG changes of hyperkalemia Calcium gluconate, sodabicarb, asthalin nebulisation, insulin Kayexelate resin. 4 marks
  • 223. What disease does this PBS picture suggest ?
  • 226. Case pH 7.51 (7.35-7.45) This is the ABG of a 6 week PO2 12 KPa (95 mmHg) neonate admitted with PCO2 4.7 KPa (35 mmHg) projectile vomiting Blood Urea 11 mmol/l Sodium 131 mmol/l Potassium 3 mmol/l Chloride 83 mmol/l
  • 227. Questions 1. What type of alkalosis the patient has: metabolic/ respiratory? 2. What is the most likely diagnosis? 3. What are the ultrasonographic criteria to diagnose the condition? 4. What is the treatment of choice?
  • 228. Answers 1. Metabolic alkalosis 2. Congenital hypertrophic pyloric stenosis 3. Ultrasonographic diagnostic criteria a) Pyloric thickness>4mm or b) Pyloric length > 14mm 4. Pyloromyotomy (Ramstedt)
  • 229. This is blood glucose profile of a 10 year boy with Diabetes. His Hb A1C is 12%. At that time his treatment included: Insulin Morning Evening Regular 6U 3U NPH 8U 5U 400 Blood glucose profile Blood Glucose (Mg/dl) 350 300 250 200 150 100 50 0 t AM AM AM 00 00 00 gh 00 00 00 12 17 22 12 17 22 ni 00 00 00 id M 6: 6: 6: TIME 2
  • 230. Questions 1. What changes would you institute in insulin dosage? 2. Considering his Hb A1c over what period his glycaemic control is likely to be unsatisfactory? 3. When do you start screening for diabetic retinopathy ? 4. What is the preferred method for screening for diabetic retinopathy?
  • 231. Answers 1 1. Increase morning regular insulin dose by 2 Units 1 2. 3 months 3. After 5 years in 1 prepubertal children after 2 years in 1 pubertal children 1 4. Fundal photography
  • 232.
  • 233. D/D of polyuria and polydipsia
  • 235. 1. What is the diagnosis? 2. What is the preferred treatment and dose? 3. Give three alternative drugs for treatment.
  • 236. Answers 1 1. Ventricular tachycardia 1 2. Cardioversion 0.5 to 1 joule/kg 3. Amiodarone 1 Procainamide 1 Lidocaine 1 Park: Pediatric Cardiology for Practitioners, 5th ed
  • 237.
  • 238. How does SVT in children differ from physiologic sinus tachycardia? SVT typically has the following features: Sudden onset and termination rather than a gradual change in rate Persistent ventricular rate of >180 bpm Fixed or almost fixed RR interval on ECG Abnormal P-wave shape or axis or absent P waves Little change in heart rate with activity, crying, or breath holding
  • 239. Case A 5 year girl is brought after accidental ingestion of unquantified insecticide with salivation, lacrimation and miosis
  • 240. Questions 1. Name three muscarinic actions of OP poisoning 2. What is the specific antidote?
  • 241. Answers 3 1. Increased oral and tracheal secretions, miosis, salivation, lacrimation, urination, vomiting, cramping, defecation, and bradycardia 2. Atropine 1
  • 242. OP poisoning effects Muscarinic effects: Increased oral and tracheal secretions, miosis, salivation, lacrimation, urination, vomiting, cramping, defecation, and bradycardia; may progress to frank pulmonary edema Central nervous system effects: Agitation, delirium, seizures, and/or coma Nicotinic effects: Sweating, muscle fasciculation, and, ultimately, paralysis
  • 243. Case A 8 year old boy is admitted after consumtion of > 60 tablets of ferrous sulfate kept at home
  • 244. Questions 1. Which stage of iron toxicity is free of any symptoms? 2. What is the preferred method of gastrointestinal decontamination in patients with iron overdose? 3. What is the antidote with dose? 4. What metal intoxication can mimic Kawasaki disease?
  • 245. Answers 1 1. Stage 2 (6-24 hours): Iron silently accumulates in the mitochondria during this period, which is relatively free of symptoms. 2. Whole-bowel irrigation -Syrup of ipecac is no longer recommended, and activated charcoal will 1 not adsorb iron. Many adult-strength iron-containing pills are very large and are often too large for orogastric lavage 3. Deferoxamine 1 Up to 15 mg/kg per hour via continuous IV infusion 1 4. Mercury 1 Acrodynia
  • 246. Iron Toxicity Stages Stage 1 (0.5-6 hours): During this stage, iron exhibits a direct corrosive effect on the small bowel. Symptoms include nausea, vomiting, abdominal pain, and/or gastrointestinal hemorrhage. Stage 2 (6-24 hours): Iron silently accumulates in the mitochondria during this period, which is relatively free of symptoms. Stage 3 (4-40 hours): This phase is characterized by systemic toxicity with shock, metabolic acidosis, depressed cardiac function, and hepatic necrosis. Stage 4 (2-8 weeks): During this phase, pyloric stenosis and obstruction can develop as a result of earlier local bowel irritation.
  • 247. Disease Present Absent a b Positive Test c d Negative •Sensitivity •Positive predictive value • T(+)/ D(+) = a/a+c •D+/T+ = a/a+b •Specificity •Negative predictive value • T(-)/ D (-) = d/b+d • D-/ T- = d/ c+d
  • 248. Case Analyse the table below Test Has disease No disease Positive 90 10 Negative 10 90 1. Calculate sensitivity 2. Calculate specificity
  • 249. Answer 1 1. Sensitivity 90% 1 2. Specificity 90%
  • 250. Case Urine Pure growth Multiple growth microscopy On auditing urine growth, following >50 WBC 95 15 were the result <50 WBC 10 200 1. Calcute positive predictive value of urine microscopy 2. Calculate negative predictive value urine microscopy
  • 251. Answer 1 1. PPV 95/110 1 2. NPV 200/210
  • 252. Case 8 A 13-year old boy develops right upper-quadrant pain and fever with chills and rigors. An abdominal ultrasound reveals Hyperechoic liver parenchyma Dilatation of several intrahepatic bile ducts Few bilateral renal cysts.
  • 253. Questions 1. What is the Diagnosis? 2. What is the cause of fever with chills in this case? 3. What are the components of this disease/syndrome? 4. Which gene has been implicated for the above?
  • 254. Answers 1 1. Caroli’s syndrome 1 2. Cholangitis 1 3. Intra-hepatic ductal ectasia AR Polycystic kidney Disease 1 4. PKHD1 gene 1
  • 255. Case9 A 3 year old boy a case of steroid sensitive nephrotic syndrome in remission has been brought with Periorbital puffiness Proteinuria on dipstick evaluation at home
  • 256. Questions 1. What are the components of nephrotic syndrome? 2. How do you define proteinuria of nephrotic range on dipstick? 3. How do you define relapse? 4. What is the treatment of relapse?
  • 257. Answers 1. Nephrotic Syndrome 0.5  Heavy proteinuria 0.5  Hypoalbuminemia 0.5  Edema  Hypercholesterolemia 0.5
  • 258. Answers 1 2. Urine protein 3+/4+ (on dipstick) 3. Urine albumin 0.5  3+ or 4+ (or proteinuria >40 mg/m2/h) 0.5  for 3 consecutive early morning specimens  having been in remission previously 0.5 0.5 2. Prednisolone  2 mg/kg/day (single or divided doses) until urine protein is 0.5 trace or nil for 3 consecutive days  Then 1.5 mg/kg on alternate days for 4 weeks  Then discontinued 0.5 0.5
  • 259. Case10 A 12 year old girl a case of chronic kidney disease due to FSGS is evaluated in OPD for hypertension, poor growth and dyslipidemia.
  • 260. Questions 1. How do you determine GFR using serum creatinine with Schwartz formula? 2. What is Stage II Chronic Kidney disease (CKD) in terms of GFR? 3. State True/ False a. Growth hormone is a mode of treatment for short stature due to CKD. b. Anemia in CKD is caused primarily due to loss of ferritin in urine. c. The most common form of renal osteodystrophy seen with CKD is high turnover bone disease.
  • 261. Answers 1. GFR(ml/min/1.73m2) 1 = k x Height (cm) serum creatinine (mg/dL) where k is  0.33 for low-birthweight infants younger than 1 yr,  0.45 for term AGA infants younger than 1 yr,  0.55 for children and adolescent females  and 0.70 for adolescent males
  • 262. Answers 1 2. Stage 2 CKD: GFR 60-89 ml/kg/1.73m2 3. True/ False 1 a. True Children with CKD who remain <2 SD for height despite optimal medical support may benefir from rHuGH (0.05mg/kg/d) b. False 1 Anemia is primarily the result of inadequate erythropoietin production c. True The most common finding in renal osteodystrophy is osteitis fibrosa cystica 1 (high turnover bone disease secondary to hyperparathyroidism)
  • 263. Answers 2. Stage 2 CKD: GFR 60-89 ml/kg/1.73m2 1 3. True/ False a. True 1 Children with CKD who remain <2 SD for height despite optimal medical support may benefir from rHuGH (0.05mg/kg/d) b. False Anemia is primarily the result of inadequate erythropoietin production 1 c. True The most common finding in renal osteodystrophy is osteitis fibrosa cystica (high turnover bone disease secondary to hyperparathyroidism) 1
  • 264. What are the main causes of chronic renal disease in children that result in renal transplantation? Obstructive uropathy Aplastic/hypoplastic/dysplastic kidneys Focal segmental glomerulosclerosis
  • 265. Questions 1. Define Time constant 2. Two conditions with increased time constant 3. Two conditions with decreased time constant
  • 266. Answers 1 1.Time constant is the product of compliance and resistance(amount of time required for proximal airway pressure to equilibrate with alveolar pressure) 2.Asthma and bronchiolitis 3.Pneumonia and pulmonary edema 1 1 Nelson18th edition pg 1722
  • 267. Questions 1. Mechanism of grunting 2. Significance of grunting 3. Helpful in which conditions (name two)
  • 268. Answers 1 1. Produced by expiration against a partially closed glottis expiration 1 2. Attempt to maintain positive airway pressure during expiration for as long as possible. 3. Alveolar diseases(pneumonia/pulmonary edema/HMD) and small airway 1 obstruction(bronchiolitis) Nelson18th edition pg 1724
  • 269. Questions 1. Significance of clubbing 2. Two thoracic causes of clubbing 3. Two extrathoracic causes of clubbing 4. Sign associated with clubbing
  • 270. Answers 1 1. Chronic hypoxia 1 2. Small lung cancer and lung abscess 1 3. Cyanotic heart diseases and IBD 4. Schamroth’s sign 1
  • 271. Case The EEG record is from a 5 month infant who episodically raises his arms and then flexes his neck, trunk and hips. The episodes last a few seconds and end with a brief cry and return to normal posture. The episodes occur in quick succession with several hours passing between each cluster of attacks.
  • 272. EEG
  • 273. Questions 1. What does the EEG show? 2. What syndrome is suggested by the history and EEG? 3. Give one genodermatoses condition associated with this syndrome. 4. What is the long term prognosis?
  • 274. Answers 1 1. Hypsarrhythmia  Large amplitude slow wave activity mixed with multifocal spikes and sharp waves
  • 275. Answers 1 2. Infantile spasms- West Syndrome 1 3. Tuberous Sclerosis 4. Treatment 1  Unsatisfactory on most occasions  AED`s Vigabatrin Benzodiazepines (Clonazepam/ Nitrazepam)  Steroids/ ACTH
  • 276. What is the classic triad of infantile spasms? Spasms, hypsarhythmia, and developmental regression Infantile spasms are known as West's syndrome, and the condition is named for the physician who first described the condition in his own son in 1841.
  • 277. After what period can AEDs be safely discontinued? When the child is free of seizures for 2 years Smith R, Ball R: Discontinuing anticonvulsant medication in children. Arch Dis Child 87:259-260, 2002
  • 278. Case 15 The examination of a newborn reveals a lump of soft tissue of size of 50 paise coin overlying the lower spine. There is no neurological deficit.
  • 279. Questions? 1. What is the likely diagnosis? 2. What is likely risk of recurrence of disorder in future pregnancies? 3. What intervention started at what time and for how long, can reduce the risk of intervention? 4. What are the options for managing bladder incontinence in this condition?
  • 280. Answers dysraphism 1. Occult spinal 1 2. Risk of recurrence 1 One affected child 3-4% Two affected Children 10% 3. Intervention Folic acid 400 mcg/d 1 Started Periconceptionally 0.5 (started before pregnancy) Continued Till 12 wks of preg 0.5
  • 281. Answers 2 4. Bladder incontinence management  Clean intermittent catheterization  Artificial urinary sphincter  Surgical urinary diversion  Augmentation cystoplasty
  • 282. What is the likelihood that a patient with myelomeningocele will have hydrocephalus? Hydrocephalus is seen in 95% of children with thoracic or high lumbar myelomeningocele. The incidence decreases progressively with more caudal spinal defects to a minimum of 60% if the myelomeningocele is located in the sacrum.
  • 283. Case 16 A 6-month-old child was noted to be normal at birth, but over the ensuing months you have been somewhat concerned about his slowish weight gain and his mild delay in achieving developmental milestones. The family calls you urgently at 7:00 A.M. noting that their child seems unable to move the right side of his body.
  • 284. Questions? 1. Which of the following conditions might explain this child’s condition? a. Phenylketonuria b. Homocystinuria c. Cystathioninuria d. Maple syrup urine disease e. Histidinemia 1. Which one other systemic examination would you like to do? 2. What investigation would confirm the etiology? 3. What treatment would you institute as a long term measure?
  • 285. Answers 1 1. Homocystinuria (b) 1 2. Eye (Ectopia lentis) 3. Elevated plasma levels for homocystine 1 4. Vitamin B6 (200-1,000 mg/24 hr) 1 Folic acid 1-5mg/24 hr Vit B 12 (1-2 g/24hrs)
  • 286. Match the following Match the following sign/symptom with the expected location of the lesion in the brain:
  • 287. a. Deafferented pupil  Nystagmus. b. Cerebellum  Tonic deviation of the eye(s). c. Ipsilateral cortex  Marcus Gunn pupil. d. Midbrain tectum  Horner’s syndrome. e Pons  Midposition pupils f. Cribriform plate of ethmoid  Pinpoint pupils  Leakage of CSF from nose.
  • 288. Answer a. Deafferented pupil Marcus Gunn pupil b. Cerebellum Nystagmus c. Ipsilateral cortex Tonic deviation of the eye(s) d. Midbrain tectum Midposition pupils e. Pons Pinpoint pupils f. Cribriform plate Leakage of CSF from nose 3 marks
  • 289. Match the following Thrombotic stroke. a. Rete mirabile of Moyamoya. b. Right-to-left cardiac shunt. Embolic stroke. c. Arteriovenous malformations. Intracerebral hemorrhage.
  • 290. Answer a. Rete mirabile of Thrombotic stroke Moyamoya. b. Right-to-left cardiac Embolic stroke shunt. c. Arteriovenous Intracerebral h`age malformations.
  • 291. Case19 A 4-year-old child is observed to hold his eyelids open with his fingers and has drooping of eyes, especially in the evening. He has some trouble swallowing his food. He can throw a ball, and he runs well. He undergoes EMG and is diagnosed as MYASTHENIA GRAVIS
  • 292. Questions? 1. What is the characteristic EMG in Myasthenia? 2. What is the chest X-ray finding in this condition? 3. What is the clinical test for diagnosing myasthenia? 4. Which antibiotics can worsen the condition? 5. What drugs are used for treatment?
  • 293. Answers 1 1. Decremental response to repetitive stimulation 2. Enlarged thymus 1 3. Edrophonium test 1 4. Aminoglycosides 5. Neostigmine 0.4mg/kg PO 4-6 hrly 1 0.04mg/kg IM 4-6 hrly 0.5 0.5
  • 294. Fleisher GR, Ludwig S (eds): Textbook of Pediatric Emergency Medicine, 3rd ed.
  • 295. Case 20 Examination of the cerebrospinal fluid of an 8-year- old, mildly febrile child with nuchal rigidity and intermittent stupor for 3 weeks shows the following: White blood cells 100/μL (all lymphocytes) Negative Gram stain Protein 750 mg/dL Glucose 25 mg/dL
  • 296. Questions? 1. What are the likely differential diagnosis? 2. What drugs can be used to reduce intracranial tension? 3. Which cranial nerve is involved in false localizing sign?
  • 297. Answers 1 1. Tubercular/ Fungal meningitis 1 2. Mannitol, glycerol 1 3. Sixth cranial Nerve (Abducens)
  • 298. Case 21 At birth, an infant is noted to have an abnormal neurologic examination. Over the next 1-2 weeks he develops severe progressive central nervous system degeneration, an enlarged liver and spleen, macroglossia, coarse facial features, and a cherry-red spot in the eye.
  • 299. Question 1. The laboratory finding likely to explain this child’s problem is a. Reduced serum hexosaminidase A activity b. Deficient activity of acid beta-galactosidase c. A defective gene on the X chromosome d. Complete lack of acid alpha-galactosidase activity e. Deficient activity of galactosyl-3-sulfate-ceramide sulfatase 1. What is the diagnosis?
  • 300. Answers 1 1. Deficient activity of acid beta-galactosidase (b) 1 2. Type 1 GM1 gangliosidosis
  • 301. 22 For each description select the most likely diagnosis:
  • 302. Clinical scenario 1. Eye blinking or throat-clearing noises in an otherwise healthy 8-year old Diagnosis boy 2. A 6-year-old boy with eye twitching a. Transient tic disorder of and ecolalia childhood 3. A 2-year-old infant who was born prematurely and is unable to walk or b. Tourette syndrome talk c. Cerebral palsy 4. A 14-year-old girl with a history of precocious puberty who now d. Tuberous sclerosis develops a large goiter e. McCune-Albright syndrome 5. An infant with infantile spasms, a hypsarrhythmic EEG pattern, and ash-leaf depigmentation on her back
  • 303. Answer 1. Transient tic disorder of childhood 2. Tourette syndrome 3. Cerebral palsy 4. McCune-Albright syndrome 5. Tuberous sclerosis Total 5 marks
  • 304. Case scenario 23 16 year boy followed up in your clinic for several years. His latest lung function tests are FVC 85% FEV1 57% PEF 53% FEV1/FVC 67%
  • 305. Questions 1. Diagnosis 2. Which of the above measurement is best for monitoring
  • 307. List the possible acute side effects of salbutamol General: Hypoxemia, tachyphylaxis Renal: Hypokalemia Cardiovascular: Tachycardia, palpitations, premature ventricular contractions, atrial fibrillation Neurologic: Headache, irritability, insomnia, tremor, weakness Gastrointestinal: Nausea, heartburn, vomiting
  • 308. What proportion of childhood asthmatics "outgrow" their symptoms? 30-50% Sears MR, Greene JM, Willan AR, et al: A longitudinal, population-based, cohort study of childhood asthma followed to adulthood. N Engl J Med 349:1414-1422, 2003.
  • 309. Question 24 Below is diagram of lung volumes,what do the letters A and B represent
  • 310.
  • 311. Answer A inspiratory reserve volume B residual lung volume
  • 312. Case No 28 7 years male c/o increasing weakness of all limbs x 3 days & URTI one week back Examination CNS Bilateral facial nerve palsies Power Grade 0 to 1 both legs; grade 3 arms Loss of DTJ,No sensory deficit/ papilloedema Abdomen Palpable midline mass arising from pelvis Other systems: Normal
  • 313. Questions 1. What is the likely diagnosis? 2. Mention 3 investigations to clinch diagnosis. 3. What is the most sensitive measure of respiratory muscle involvement? 4. Mention 3 modalities of treatment. 5. What is the midline mass, explain its significance and how should it be managed?
  • 314. Answers 1. Guillain-Barre Syndrome (1)  The history points to a symmetrical motor neuropathy without sensory nerve involvement.  Muscle disorders should have normal DTJ  Polio will cause asymmetrical weakness  Spinal cord abnormalities will not cause facial muscle weakness
  • 315. Answers 2. CSF-protein cellular dissociation NCV EMG (1.5) 3. The most sensitive measure of respiratory muscle involvement is (1) VITAL CAPACITY In children, the normal VC may be calculated as VC = 200 mL × age in years. If the VC falls below 25% of normal, endotracheal intubation is performed. 4. IVIG,Plasmapheresis,steroids (1.5) 5. Bladder distension due to ANS involvement (1)
  • 316. Case No 30 A 2 years boy was unconscious for 1 min following head injury. Although he is fully alert now there is bruising on the left side of the head over the parietal bone. Skull x-rays are performed and he is admitted for neurological observations.
  • 317. Questions 1. What clinical features would suggest requirement of CT skull and/ or neurosurgical opinion? 2. What non-surgical temporary measures can be used to reduce raised intracranial pressure? 3. What advice needs to be given to parents if after 24 h the child has remained well and is ready for discharge?
  • 318. Answers 1. Neurosurgical Opinion/ CT skull a. Deteriorating level of consciousness b. Focal neurological signs c. Depressed Skull fracture d. Basal Skull fracture e. Seizures f. CSF leak g. Signs of Raised ICT (2)
  • 319. Answers 2. Temporary manoeuvres to reduce raised ICT  Nursing in a 30 degrees head up position  Diuretics (Mannitol)  Hypertonic saline  Artificial hyperventilation (2)
  • 320. Answers 3. At discharge it should be stressed that child should be brought back if he develops any of following:  Vomiting  Drowsy/ altered sensorium  Blurred/ double vision  Seizures (2)
  • 321. CASE SCENARIO 7 year old girl presents with high irregular fever for 3 weeks, each episode associated with erythematous rashes over the trunk and proximal extremities, pain and swelling of Rt knee and easy fatigability. Has leucocytosis, raised ESR, normal chest skiagram and negative Mx. 1. Give most probable diagnosis. 2. List other expected systemic clinical findings 3. Give value of serology in diagnosis 4. Outline steps in management 5. List prognostic factors
  • 322. 1. JRA – Systemic 0.5 2. Systemic exam findings a. Hepato – splenomegaly 0.5 b. Lymphadenopathy 0.5 c. Serous cavity effusions 0.5 3. Serology a. ANA - Negative 0.5 b. RF - Negative 0.5 4. Management a. “Pyramid” approach – NSAIDS, chloroquine, 1.0 methotrexate,azothiaprine/cyclophosphamide. b. Steroids 0.5 c. TNF-alpha receptor antagonists 0.5 5. Number and severity of joint involvement 1.0 (Total Marks 6.0)
  • 323. This patient becomes ill with thrombocytopenia, profound anemia, and markedly elevated transaminases probably has what complication? Macrophage activation syndrome
  • 324. CASE Scenario 12 yr old boy, case of Haemophilia A, is admitted with spontaneous haemarthrosis. 1. What is the level of factor VIII ? 2. Factor VIII in treatment. a. Level of factor VIII to be achieved b. Dose of factor VIII 3. The patient has to undergo major surgery a. Level of factor VIII to be achieved b. Dose of factor VIII 4. What are the other precautions? 5. What is the role of DDAVP in haemophilia A ?
  • 325. 1. <1% 0.5 2. Factor VIII in treatment a. 35-40% 1.0 b. 20 units/kg, repeat daily if required 1.0 till joint normalises 3. Dose in major surgery a. 100% 1.0 b. 50 units/kg, infuse 2-3 units/kg/hr to 1.0 maintain level at 100%for 24 hrs and then >50% for 7 days 4. Precautions a. Avoid Anti-inflammatory drugs with anti-platelet 0.5 action b. Screen for Transfusion transmitted diseases if 0.5 plasma products are used 4. Desmopressin Acetate: Release of factor VIII in mild cases 0.5 (Total marks 6.0)
  • 326. CASE Scenario A 7 yr old girl with strong family history of chronic fatal liver disease presents with features of haemolysis. 1. List one clinical examination which may give the diagnosis 2.State most probable group of aetiology based on answer to question ‘1’ 3.List three most important tests for diagnosis with values 4.Outline the management of this child
  • 327. 1. Kayser-Fleischer rings 0.5 2. Wilson disease 0.5 3. Diagnostic tests a.Serum ceruloplasmin 0.5 < 200 mg/lit 0.5 b. Urinary copper 0.5 >100 microgram/day 0.5 c. Hepatic copper 0.5 >250 microgram/gm of dry weight 0.5 1. Management a. Restrict copper intake 0.5 b. Chelation with penicillamine 0.5 c. B6 supplementation 0.5 d. Liver transplant 0.5 (Total marks 6.0)
  • 329. Identify the instrument. Which illness is it used in ? What are the three settings ?
  • 330. Peak Flow Meter Bronchial Asthma 1. Green zone >80% Peak Exp Flow 2. Yellow Zone 50-80 % PEF 3. Red zone < 50% PEF Total 3 marks
  • 331. Which class does this antibiotic belong to ? What is the spectrum? What is the dosage? What is the imp clinical use? 4 marks
  • 332. Oxazolidinones Gram-positive organisms including MRSA Penicillin-resistant pneumococci Gram-negative anaerobes Mycobacteria 10mg/kg/dose 8 hrly Multiresistant gram-positive organisms, including MRSA who have failed to respond to vancomycin.
  • 333. Indications for use of this compound? what is the dose? What are the indications When should it be started? What are the side effects of parenteral nutrition? 4 marks
  • 334. Parental nutrition Dose 0.5 -3.5 g/kg/day When enteral nutrition is insufficient or contra- indicated. Day 1 Dyselectronemia, cholestasis, bacterial or fungal colonisation and sepsis, thrombosis, azotemia, thrombocytopenia
  • 335. Identify the product? What are the advantages of its use? Enumerate five complications?
  • 336. PICC 1 Preserves veins, allows concentrated infusates, permits long term IV alimentation 1 6 complications- 3 Catheter fracture and embolisation, Leakage  Thrombosis of catheter Accidental displacement / Tip Migration Deep Venous Thrombosis Infection
  • 337. What is the equipment ? Identify the parts. Enumerate indications for its use. What are the contra- indications? How do you sterilise it? 5 marks
  • 338. Self inflating 250 ml ambu bag 1 Oxygen , air inlet , pt outlet, valve assembly, oxygen reservoir and pressure release valve. 1 Gasping/apneic infant, cyanosis inspite of free flow oxygen and HR<100/m 30secs after initial steps. 1 MSAF and suspected cong diaph hernia 1  Wash with detergent and hot water. Dry thoroughly. If visibly soiled with blood/body fluids, clean and send to Central Sterilization Dept 1
  • 339. Identify the product? What is the dosage? What are the indications for use? Enumerate three precautions ?
  • 340.  Surfactant 0.5  4ml/kg 0.5  Surfactant replacement therapy for prevention and treatment of RDS in preterms, 0.5  Survanta should not be removed from the refrigerator for more than 24 hours and it should not be warmed and returned to the refrigerator more than once.  Do not suction the infant for 1 hour after dosing unless signs of significant airway obstruction occur.  Marked improvements in oxygenation and decrease in lung compliance may occur within minutes of administration of SURVANTA. Therefore, frequent and careful clinical observation and monitoring.  Donot shake the vial 1.5
  • 341. What is the equipment? What are the indications of its use? What are the advantages?
  • 342. Double lumen endotracheal tube 1 For surfactant instillation 1 To aid lung lavage Permits efficient drug instillation without interrupting ventilation 1
  • 343.
  • 344. Identify the equipment Indications for usage Complications (mention any three)
  • 345.  Bubble CPAP apparatus 1  Indications - 2 • If FiO2 requirement is > 0.3 • SA Score > 4 • ABG Score > 3 on oxygen therapy • Frequent apnea with documented hypoxemia • Clinically significant chest retractions after extubation from mechanical ventilation • Complications 1 • Air leak • Reduced venous return • IVH
  • 346. Which class does this antibiotic belong to ? What is the spectrum? What is the dosage? What is the two imp side effects?
  • 347. Carbapenems Widest spectrum of antimicrobial activity among the β-lactam antibiotics against both gm-ve and gm + organisms except MRSA. 20 - 40mg/kg/dose 12hrly Transient agranulocytosis, thrombocytopenia. Seizures 2 marks
  • 348.
  • 350. What are the- Abnormalities? 1 Diagnosis? 1 Likely causes? 1
  • 351. Low pH, high PaCO2, low PaO2, high HCO3 Uncompensated respiratory acidosis with hypoxemia RESPIRATORY FAILURE (pneumonia, RDS)
  • 352. ABG 2 pH 7.57 PaCO2 22mmHg PaO2 156mmHg HCO3 18mEq/L Base excess -8
  • 353. What are the- Abnormalities? 1 Diagnosis? 1 Likely cause? 1
  • 354. High pH, low PaCO2, high PaO2, low HCO3 UNCOMPENSATED RESPIRATORY ALKALOSIS WITH HYPEROXIA Hyperventilation with high FiO2
  • 355. ABG 3  pH 7.32  PaCO2 30mmHg  PaO2 70mmHg  HCO3 12mEq/L  Base excess -8
  • 356. QUESTIONS  ABNORMALITIES 1  DIAGNOSIS 1  LIKELY CAUSES 1
  • 357. • Low pH, low PaCO2, normal PaO2, low HCO3 • UNCOMPENSATED METABOLIC ACIDOSIS • SHOCK, RENAL FAILURE
  • 358. ABG 4  pH 7.60  PaCO2 21mmHg  PaO2 65mmHg  HCO3 24mEq/L  Base excess +2
  • 359. QUESTIONS  ABNORMALITIES 1  DIAGNOSIS 1  LIKELY CAUSES 1
  • 360. High pH, low PaCO2, normal PaO2, normal HCO3 UNCOMPENSATED RESPIRATORY ALKALOSIS HYPERVENTILATION (BRONCHIAL ASTHMA)
  • 361. ABG 5  pH 7.36  PaCO2 70mmHg  PaO2 75mmHg  HCO3 35mEq/L  Base excess +14
  • 362. QUESTIONS  ABNORMALITIES 1  DIAGNOSIS 1  LIKELY CAUSES 1
  • 363. Low pH, high PaCO2, normal PaO2, high HCO3 COMPENSATED RESPIRATORY ACIDOSIS VENTILATED INFANT WITH TUBE BLOCK
  • 364. ABG 6  pH 7.52  PaCO2 47mmHg  PaO2 80mmHg  HCO3 30mEq/L  Base excess +3
  • 366. High pH, high PaCO2, normal PaO2, high HCO3 UNCOMPENSATED METABOLIC ALKALOSIS VOMITING, PYLORIC STENOSIS
  • 367. ABG 7  pH 7.14  PaCO2 54mmHg  PaO2 55mmHg  HCO3 14 mEq/L  Base excess -7
  • 368. QUESTIONS  ABNORMALITIES 1  DIAGNOSIS 1  LIKELY CAUSES 1
  • 369. Low pH, high PaCO2, normal PaO2, low HCO3  UNCOMPENSATED RESPIRATORY AND METABOLIC ACIDOSIS  INFANT ON VENTILATOR WITH TUBE BLOCK & SHOCK
  • 371. Brain Stem cell glioma, 1 mark
  • 372. Observed Station 1 DRUG COUNSELLING 4 yr old boy weighing 15 Kg , diagnosed as Nephrotic Syndrome to be started on steroids.
  • 373. 1. Introduces self/ puts the child and attendant at ease. 0.5 2. Explains the disease in simple words 0.5 3. Explains the medication  Dose 0.5  Frequency 0.5  Relationship with meals 0.5 4. Explains side effects  GIT 0.5  Steroid effect 0.5 5. Explains monitoring of response  Urinary output 0.5  Body weight 0.5 6. Asks for queries if any. 0.5 7. Advises to report back if any problems. 0.5 8. Advises about pain abdomen 0.5 (Total marks 6.0)
  • 374. Observed Station 2 DRUG COUNSELING A 8yr old boy, weighing 30 kg, diagnosed as DM Type I requires 30 units insulin/day, in two divided doses ,combining short and intermediate acting insulins. Counsel regarding administration.
  • 375. 1.Introduces self & puts child and 0.5 attendant at ease. 2. Explains the problem in brief. 0.5 3. Familiarizes the attendant with the injectables 0.5 and the syringes. 4. Explains dose, time of injection and relationship 2.0 with meals, sites, route and rotation of sites 5. Explains the calculation of dose in ml. 1.0 6. Explains the sequence of loading (regular first). 0.5 7. Briefs about hypoglycemia incl mgt. 0.5 8. Inquires about any doubts and advises to report 0.5 back in case of any problems. (Dose – 2/3rd before breakfast,1/3rd before dinner) (Ratio intermediate /regular 2:1/3:1) (Total marks 6.0)
  • 376. Observed Station 3 COUNSELING 24 yr old lady diagnosed as HIV + at 36 weeks of gestation. Counsel regarding perinatal transmission and follow up.
  • 377. 1. Ensures the presence of husband 0.5 2. Introduces self/ puts the couple at ease. 0.5 3. Explains the disease in simple words 0.5 4. Explains the incidence and modes of perinatal transmission 20-30% 0.5 Prenatal 0.5 Intranatal 0.5 Breast feeding 0.5 5. Explains modalities of reducing rate of transmission ART to mother and child 1.0 LSCS Vs Vaginal delivery 1.0 Breast feeding Vs top feeding 1.0 6. Explains effect of measures – reduction by 50% 1.0 7. Explains screening of the infant 0.5 8. Explains safety of cuddling, petting and kissing 1.0 9. Asks for queries if any. 0.5 10. Advises to report back if any problems. 0.5 (Total marks 10.0)
  • 378. Observed Station 4 Drug Counseling 4 yr old child (15Kg) ,a case of gen tonic-clonic seizures has been advised syr carbamezapine (100 mg/5ml).Counsel the mother regarding administration.
  • 379. 1. Introduces self & puts child and 0.5 attendant at ease. 2. Explains the problem in brief. 0.5 3. Familiarizes the attendant with the drug 0.5 and dispenser 4. Explains dose(75 mg-3.5 ml), in 3 divided 0.5 doses 5. Advises not to miss the dose 0.5 6. Increase to (150 mg – 7.5 ml) 0.5 7. Explains side effects 1.0 8. Calls for blood counts weekly 0.5 9. Calls for serum levels of the drug 1.0 10. Advises to report in case of doubt 0.5 (Total marks 6.0)
  • 380. Observed Station 5 DRUG COUNSELING 2 yr old child weighing 10 Kg is being discharged after admission for febrile seizure. Counsel the mother regarding domiciliary management of seizure with rectal diazepam.
  • 381. 1. Introduces self & puts child and 0.5 attendant at ease. 2. Explains the problem in brief. 0.5 3. Familiarizes the attendant with the drug, 0.5 the syringe and the catheter. 4. Explains dose(3mg), and loading of the syringe 0.5 5. Explains the position of the child (lateral) 0.5 6. Explains introduction of the catheter (lubrication, 1.0 and length of introduction) 7. Explains pushing the drug 0.5 8. Advises pinching of the buttocks together after 1.0 9. Withdrawing the catheter) 10. Advises one repetition after 15 mts 0.5 11. Explains side effects 0.5 (Total marks 6.0)
  • 382. http://groups.yahoo.com/group/PediatricsDNB/ oscepediatrics.blogspot.in/

Notas del editor

  1. Gradrng of vesicoureteral reflux. Grade I: reflux into a nondilated ureter. Grade II: reflux into the upper collecting system withour dilatation. Grade III: reflur into dilated ureter and/or bluntinu of calvceal fornices. Grade IV: reflux into a grossly dilated ureter. Grade V Massive reflux. With significant ureteral dilatation and tortuosity and loss of the papillary impression
  2. Acrodynia The symptom complex included swelling and redness of the hands and feet, skin rashes, diaphoresis, tachycardia, hypertension, photophobia, and an intense irritability with anorexia and insomnia. Infants were often very limp, laying in a frog-like position, with impressive weakness of the hip and shoulder girdle muscles. Similar symptoms have been described in children exposed to other forms of mercury, including broken fluorescent light bulbs or diapers rinsed in mercuric chloride.